Работна тетратка по математика

Page 1

Rabotna tetratka po

MATEMATIKA za VII oddelenie



PREDGOVOR Pri izu~uvaweto na matematikata vo VII oddelenie ti pomaga u~ebnikot po matematika od koj mo`e{ da razbere{ i nau~i{ mnogu novi poimi, kako i da se potseti{ na ve}e izu~enite, da se zapoznae{ so mnogu pravila {to ti ovozmo`uвaat da navleze{ vo tajnite na matematikata. Rabotnata tetratka po matematika za VII oddelenie, kako sostaven del na u~ebnikot ti pomaga da gi prodlabo~i{, pro{iri{ i proveri{ so kakvi i kolkavi znaewa si se steknal vo nastavata. Rabotnata tetratka isto kako i u~ebnikot e podelena na пет delovi soglasno temite so nastavnata programa po matematika za VII oddelenie. Taa sodr`i: • Tematski ve`bi za sekoja od nastavnite sodr`ini so po 10 zada~i (nekade i pomalku) zavisno od obemnosta na sodr`inata. • Testovi za samoproverka za sekoja tema, {to se dadeni na krajot od sekoja tema. Во тестовите од тема I, тема IV и тема V, prvite 5 pra{awa (zada~i, koi se re{avaat usno ili bez pogolemi pote{kotii), predlagame da se vrednuvaat so 6 bodovi, dodeka pak vtorite 5 zada~i od testot da se vrednuvaat so 14 bodovi. Site zada~i od testovite vo tema II se vrednuvaat so 20 bodovi, a site zada~i od testovite vo tema III se vrednuvaat so 10 bodovi. Zna~i vo sekoj test vkupniot broj na mo`ni bodovi e 100. Za pretvorawe na bodovite od testot vo ocenki ja predlagame slednava tabela: Bodovi

85 – 100

65 – 84

45 – 64

25 – 44

0 – 24

Ocenka

Odli~en (5)

Mnogu dobar (4)

Dobar (3)

Dovolen (2)

Nedovolen (1)


Tema I

VEKTORI. TRANSLACIJA

1. PRAVEC I NASOKA. NASO^ENA OTSE^KA - VEKTOR 1. Relacijata R: “pravata p e paralelna na ...” gi ima slednite svojstva: a) Pravata p e ________________________________________________________ sama na sebe. b) Ako pravata p e paralelna na pravata q, toga{ ___________________________________ . v) Ako p II q i q II r, toga{ i ________________________________________________________ .

2. a) [ to e pravec opredelen so dadena prava? b) Kolku nasoki ima daden pravec? Odgovor: a) Mno`estvoto ____________________________________________________________ ___________________________________________________________________________ b) Sekoj pravec ____________________________________________________________ ___________________________________________________________________________

3. Kolku pravi minuvaat niz: a) edna to~ka; b) dve razli~ni to~ki. Odgovor: a) Niz edna to~ka __________________________________________________________ ___________________________________________________________________________ b) Niz dve razli~ni to~ki __________________________________________________ ___________________________________________________________________________

4. Dadeni se pravite a, b, c i d, taka {to a II b, a ја сече правата c, i pravata d gi se~e trite pravi. Vo vrska so tie pravi, dadeni se nekolku iskazi. Koj od niv e to~en? a) O 1 A 2 ↑↑ O 1 A 3 b) O 1 A 3 ↑↓ O 1 A 1 v) O 1 A 1 ↑↑ O 3 C 3

g) O 2 B 1 ↑↓ O 1 A 3 __________ d) O 2 B 2 ↑↑ O 3 C 2 |) O 1 A 1 ↑↓ O 3 C 1

Odgovor: To~ni se slednite iskazi: ___________________________________________________

5. Koja otse~ka se vika vektor? Odgovor: Otse~kata _________________________________________________________________ __________________________________________________________________________ .

5


6. Izberi tri razli~ni to~ki A, B i C. Kolku vektori mo`at da se formiraat so po~etok vo edna, a krajot vo druga dadena to~ka? Kolku vkupno vektori obrazuvaat dadenite to~ki?

Odgovor: Mo`e da se formiraat ____________________ vektori. Ima vkupno ____________________ vektori.       7. Dadeni se vektorite a , b , c , d , e , f . Koi od niv se:

a) kolinearni; b) istonaso~eni; v) sprotivnonaso~eni?

Odgovor: a) Kolinearni se: __________, __________ .

b) Istonaso~eni se: __________.

v) Sprotivnonaso~eni se: __________, __________ . 

8. Vektorot a e napolno (ednozna~no) opredelen so:

Odgovor: a) ________________________________________________________________________ . b) _______________________________________________________________________ . v) ________________________________________________________________________ .

9. Neka to~kite M i N se sovpa|aat vo edna ramnina. a) Zapi{i vektor so po~etna to~ka M i krajna N; b) Zapi{i ja goleminata na toj vektor; v) Koja otse~ka se narekuva nulti vektor?

Odgovor: a) ________________________________________________________________________ . b) _______________________________________________________________________ . v) ________________________________________________________________________ .

6


2. EDNAKVOST NA VEKTORI 1. Na crte`ot se dadeni pove}e vektori: a) Istonaso~eni se vektorite _________________ . b) Sprotivnonaso~eni se vektorite ___________ . v) Ednakvi se vektorite _______________________ .

2. Koga dva vektora a i b se ednakvi?   Odgovor: Vektorite a i b se ednakvi ako:

a) _______________________________________________________________________________ . b) ______________________________________________________________________________ .

3. Za koi dva vektori m i n velime deka se sprotivni?   Odgovor: Vektorite m i n se sprotivni ako:

a) _______________________________________________________________________ . b) ________________________________________________________________________ .

4. Daden e vektorot MN ( MN = 3 cm). Prenesi go vektorot MN , taka {to to~kata M ≡ S . Kolku takvi vektori ima?

Odgovor:

__________________________________________________________________________ .

5. Daden e vektorot AB . Konstruiraj: a) Vektor MN ednakov na AB , b) Vektor EF sprotiven na AB . Odgovor: a)

b)

7


6. Daden e paralelogramot ABCD (AB II CD i AD II BC) i to~kata S, presek na dijagonalite AC i BD. a) Koi vektori se kolinearni, a ne se ednakvi? b) Koi vektori se ednakvi? v) Koi vektori se sprotivni?

Odgovor: a) _____________________________ .

b) ________________________________ .

v) _____________________________ .

7. Dadena e kru`nicata k (O,r) i na nea se ozna~eni vektorite OP i MN . Konstruiraj vektor, a) Sprotiven na vektor OP ? b) Ednakov na vektor MN ?

8. Proveri ja to~nosta na iskazot: Ako vektorot MN = EF , toga{ i vektorot ME = NF . (Napravi crte`!).

9. Ako vektorite AB = CD i AM = CN , toga{ sleduva i vektorot BM = DN . Doka`i!

Dadeno:

AB = CD i AM = CN

Tvrdime: BM = DN Dokaz:

8


3. SOBIRAWE NA VEKTORI. SVOJSTVA 1. Najdi go zbirot, konstruktivno, na kolinearnite vektori:   a) a + b ,

Odgovor: a)

   b) a + b + c

b)

2. Najdi go zbirot na vektorite AB i CD , konstruktivno, po praviloto na triagolnikot.

   3. Konstruiraj go zbirot na nekolinearnite vektori m , n i p so nadovrzuvawe i po~etna to~ka S.

4. Na {to e ednakov zbirot na vektorite: a) AB + BC + CA

b) AB + BC + CD + DA

Odgovor: a) AB + BC + CA = _______________

b) AB + BC + CD + DA = _______________

9


5. Konstruiraj go zbirot na vektorite MN i PQ po praviloto na paralelogram so po~etna to~ka S.

6. Poka`i deka za koi bilo vektori m i n va`i m + n = n + m (koristi go praviloto na paralelogram za sobirawe na vektori).

    7. Neka vektorite e i f se sprotivni. Konstruiraj go nivniot zbir e + f .

8. Daden e trapezot ABCD (ABIICD). To~kite M i N se sredini na kracite AD i BC soodvetno. Izrazi go vektorot MN so pomo{ na vektorot AB i CD (obrazlo`i go odgovorot).

Odgovor:

MN = _____________________

9. Daden e ~etiriagolnikot ABCD kako na crte`ot. Izrazi go vektorot:  b;  c;  v) AD so pomo{ na vektorite AC i c ;  g) AD so pomo{ na vektorite a i BD ;  a) AC so pomo{ na vektorite a i  b) BD so pomo{ na vektorite b i

[ to zaklu~uva{ za operacijata sobirawe vektori: a) _________________ b) ___________________ v) ___________________ g) ________________ Odgovor: Operacijata sobirawe vektori e ____________________________________________

10


4. ODZEMAWE NA VEKTORI. MNO@EWE NA VEKTORI SO BROJ   1. Opredeli go zbirot na sprotivnonaso~enite vektori a i b :         b) a + b ako a < b . a) a + b ako a > b ,

2. Opredeli ja, konstruktivno, razlikata na kolinearnite vektori m i n :         b) m − n ako m < n . a) m − n ako m > n ,

3. [ to zna~i vektorot b da se odzeme od vektorot a ?   Odgovor: Da se odzeme vektorot b od vektorot a _______________________________________

___________________________________________________________________________

4. Dadeni se nekolinearnite vektori AB i CD . Konstruiraj ja razlikata AB − CD so po~etna to~ka S.

5. Neka n e proizvolen vektor. Opredeli ja razlikata:   a) n − o

  b) n − n

    v) n − m kade m = −n

11


6. Daden e paralelogramot ABCD (ABIICD i ADIIBC) i S prese~na to~ka na dijagonalite AC i BD. Razlikata na vektorite e:

a) AB − AD = ______________ b) BC − BD = _______________ v) AB − BC = _______________ g) AS − SC = _______________

7. Vo koi slu~ai va`i ravenstvoto:     a) a + b = a + b

    b) a + b = a − b

    v) a − b = a − b

Odgovor: a) _________________________________________________________________________ b) ________________________________________________________________________ v) _________________________________________________________________________  8. Neka a e proizvolen vektor. Opredeli go, konstruktivno, vektorot:   a) m = 3a

 3 b) n = a 4

9. Opredeli go, konstruktivno, vektorot:       b) 3a − 2b ako a i b se dadeni vektori. a) 2a + 3b

12


5. TRANSLACIJA 1.

Neka vagon~eto se dvi`i pravoliniski po {inite na prugata, pri {to vo sekoja naredna polo`ba ostanuva paralelno so prvobitnata i pritoa formata i goleminata ostanuvaat postojani. Vakvoto dvi`ewe se vika paralelno pomestuvawe ili ______________________________________

2. Koe preslikuvawe se vika translacija? Odgovor:

 Translacija za vektorot a se vika

_____________________________________________________________________________________

3. So pomo{ na translacija nacrtaj: a) prava a paralelna na dadena prava p, b) prava b paralelna na dadena prava p i minuva niz dadena to~ka B.

 4. a) Pri translacija τ a , vektorot a se vika __________________________________________

to~kata M1 = τ a (M ) e _________________________________ na originalot ________________ b) Translacijata τ −a se narekuva _________________________ za translacijata _________

5. Dadeni se nekolinearnite to~ki A, B i C. Konstruiraj gi slikite na ovie to~ki pri translacija τ AB .

13


6. Dadena e pravata p i to~kite M, N, P i Q od taa prava. Konstruiraj gi slikite na tie to~ki pri translacija τ . PN

τ PN (M) = _________; τ PN (N) = _________; τ PN (P) = _________; τ PN (Q) = _________.

Odgovor:

7. Dadena e otse~kata MN i vektorot n . Konstruiraj ja slikata M 1 N 1 na otse~kata MN pri translacija τ n .

τ n (MN) = _________

Odgovor:

8. Konstruiraj ja slikata na ∆ABC pri translacija τ za vektorot a .

τ a (∆ABC) = _________

Odgovor:

9. Neka a i b se dva nekolinearni vektora i M dadena to~ka. Da se konstruiraat to~kite M 1 = τ a +b (M) i M 2 = τ b + a (M). Dali to~kite M 1 i M 2 se sovpa|aat?

Odgovor:

14

To~kite M 1 i M 2 _________________________________________________________ .


6. SVOJSTVA NA TRANSLACIJATA 1. Osnovni svojstva na translacijata τ a se: a) Translacijata go zapazuva ________________________________________________________ b) Ako to~kata M le`i me|u to~kite A i B, toga{ i ___________________________________ v) Sekoja figura F pri translacija τ a se preslikuva vo _______________________________  2. Izvr{i translacija τ za vektorot a na otse~kata MN. Ako M1 = τ a (Μ ) i N1 = τ a (N ) , toga{ sporedi gi otse~kite MN i M 1 N 1 .

Odgovor:

MN __________________ M1 N1

3. Dadena e otse~kata AB i to~kata P sredina na AB. Konstruiraj ja slikata na AB pri translacija τ n . Sporedi go redosledot na to~kite A, M i B i nivnite sliki.

Odgovor: Redosledot na slikite od to~kite A, M i B e ______________________________ redosledot na to~kite A, M i B.  4. Daden e ∆ABC i vektor a . Konstruiraj ja slikata na ∆ABC pri translacija τ a . [ to mo-

`e{ da zaklu~i{ za ∆ABC i negovata slika pri translacija τ a .

Odgovor: ∆ABC i negovata slika ________________________________________________________

15


5. Nacrtaj proizvolen ostar agol, a potoa konstruiraj ja slikata pri translacija τ za dade niot vektor a .

  6. Konstruiraj ja slikata na pravata p pri translacija τ za vektorot a , ako a || p.

Odgovor: Pri ovaa translacija pravata p se preslikuva vo_______________________________ 7. Dadeni se paralelnite pravi p i q i to~kite P i Q taka {to P ∈ p i Q ∈ q . Izvr{i translacija na pravite p i q za vektorot PQ .

Odgovor: Pravata p pri τ

PQ

se preslikuva vo ____________, a pravata q vo _____________ .

8. Konstruiraj ja slikata na pravoagolnikot ABCD pri translacija τ  x = AD + DC .

za vektorot

 9. Izvr{i translacija τ na kru`nicata k (O,r = 2 sm) za vektorot а = ОP , P to~ka od kru`nicata k.

16


7. PRIMENA NA TRANSLACIJA  1. Izvr{i translacija na kru`nicata k (O,r = 2,5 cm) pri translacija τ za vektor x so dol `ina x = 2r , pravec i nasoka po tvoj izbor.

 2. Dadeni se pravata a, b i vektorot n . Konstruiraj ja slikata M 1 na pravata b pri translacija τ n na to~ka M od pravata a.

3. Dadeni se dva agla so zaemno paralelni i sprotivnonaso~eni kraci. So pomo{ na translacija doka`i deka tie se ednakvi. Dadeno: OA↑↓O 1 A 1 i OB↑↓O 1 B 1 Tvrdime: ∠AOB ≅ ∠A 1 O 1 B 1 Dokaz:

4. So pomo{ na translacija, doka`i deka sosednite agli kaj rombot se suplementni.

17


5. Konstruiraj kru`nica koja minuva niz dadena to~ka P i dopira dve paralelni pravi p i q.

6. Ako pravite a, b se zaemno normalni i ako a 1 = τ n (a) i b 1 = τ n (b), toga{ i pravite a 1 i b 1 se, isto taka, zaemno normalni. Doka`i! Dadeno: a ⊥ b i a1 = τ n (a ), b1 = τ n (b ) Tvrdime: a 1 ⊥ b 1 Dokaz:

7. Konstruiraj ramnokrak trapez ABCD (AB||CD), ako se dadeni osnovite a, b i krakot c. Dadeno

Skica

Konstrukcija

 8. Dadena e kru`nica k (0,r), pravata p i vektorot a . Konstruiraj ja to~kata M 1 na pravata p, slika na to~kata M {to le`i na kru`nicata k pri translacijata τ a .

18


STEPENI. KVADRATEN KOREN

Tema II

1. POIM ZA STEPEN SO POKAZATEL PRIRODEN BROJ. 1.

Zapi{i gi najkratko slednive izrazi: a) 4 + 4 + 4 = i 4 ⋅ 4 ⋅ 4 = b) x ⋅ x ⋅ x ⋅ x = i x + x + x + x = v)

a a a a a ⋅ ⋅ ⋅ ⋅ = 2 2 2 2 2

) ⋅ (2 ) ⋅ ..... (2 g) ( 2 x x ) ⋅ ( 2 x x ) =  10− mno`iteli

2. Zapi{i gi vo vid na proizvod slednite stepeni: a) (− 4)3 =

3.

2

 3 v)   =  4

b) (2x )4 =

g) (a − 3)5 =

Popolni ja tablicava: Stepen

2

 2    4

3

2

(− 5)

(2x )

4

 3 −   5

2

(x + 1)0

Osnova

−3

−3,1

n3

Eksponent

1

0

5

4.

(− 3)7

2 3 4

Presmetaj ja vrednosta na izrazot: a) 32 − 2 3 ;

b) 5 2 + (− 2)5 ;

v) 2 x 2 − 3x 3 ;

za x ∈ {− 2,−1,0,1,2}

29


5.

Odredi go znakot na brojot: a) (− 1)20 ;

6.

30

g) (− 4)100 .

b) 100 000;

v) 1500;

g) 74 000 000.

Zapi{i gi kratko, so pomo{ na stepen so osnova 0,1 broevite: a) 0,001;

8.

v) (− 3)3 ;

Zapi{i gi kratko, so pomo{ na stepen so osnova 10 broevite: a) 1000;

7.

b) (− 1)15 ;

b) 0,000001;

Presmetaj ja vrednosta na sekoj od izrazite: 1 a) 0,4 ⋅ 10 3 − 7 4 : 49 + 4 3 ⋅ ⋅ = 64

v) 0,0000001;

b) 200 − 8 2 : 4 3 + 10 3 =

g) 0,1.


2. MNO@EWE I DELEWE NA STEPENI SO EDNAKVI OSNOVI. STEPENUVAWE NA PROIZVOD. KOLI^NIK I STEPEN 1.

Izvr{i go mno`eweto na stepenite: b) (− 3)2 ⋅ (− 3)5 =

a) 4 2 ⋅ 4 =

g) (a − 2)0 ⋅ (a − 2) ⋅ (a − 2)2 =

v) x 4 ⋅ x ⋅ x 3 =

2. Za koj broj n ∈ Ν e to~no tvrdeweto: b) x ⋅ x 2 ⋅ x 3 = x n ?

a) 2 2 ⋅ 2 n = 2 7 ;

3.

Poka`i deka: b) izrazot 512 − 510 e deliv so 24.

a) izrazot 12 5 − 12 4 e deliv so 11;

4.

Presmetaj gi slednive koli~nici: 10

a) 3 : 3 = 4

5.

2

3

 4  4 b)  −  :  −  =  5  5

Presmetaj:

(

)

a) x 5 : x 2 ⋅ x 3 ;

(

g) (x − 3)5 : (x − 3)5 =

v) a 30 : a12 =

)

b) y10 : y 2 ⋅ y 3 ;

v)

(

a8 ⋅ a3 ⋅ a 2 a13

)

;

g)

(

)

5 4 ⋅ 55 ⋅ 5 0 . 5

31


6.

Izrazi go stepenuvaweto: a) (4xy ) ; 2

7.

3

( )

2

32

2

 2x3 y 3   . g)   5  

2

3

 7 v)  −  ;  12 

a b)   ;  2

5

 2a 3 y 4  g)  5 3  .  3a y 

Zapi{i go izrazot vo vid na stepen so osnova a: a) a 3 ;

9.

4

 1  v)  − a 3 x 2  ;   2

Stepenuvaj gi dropkite:  3 a)   ;  4

8.

b) (− axy ) ; 3

( ) ⋅ (a ) ;

b) a 4

2

2 4

Presmetaj ja vrednosta na izrazot

(

)

2

v) a 3 ⋅ a 8 ;

( )

x5 ⋅ x 2 x10

3

za x = −4,2 .

g)

(

a3 ⋅ a ⋅ a3 a7

)

2

.


3. POIM ZA KVADRAT NA RACIONALEN BROJ. POIM ZA KVADRATEN KOREN OD RACIONALEN BROJ 1.

Zapi{i gi vo vid na proizvod slednive stepeni: b) (− 3)2 ;

a) 4 2 ;

2.

3.

2

 4 g)   . 7

v) (− 0,5) 2 ;

Zapi{i gi proizvodite vo vid na kvadrat (stepen) na broj: 2 2 a) 3 ⋅ 3 ; b) (− 4) ⋅ (− 4) ; v) ⋅ ; 3 3

g) 0 ⋅ 0 .

Popolni ja tablicata: x

1 2

1 3

−1

1

0

−0,4

3 7

0,01

−3

10

2

x2

4.

Presmetaj go kvadratot na broevite: -6, 6, -2, -10, 7, -5.

5. Presmetaj ja vrednosta na izrazot: a) (− 4)2 − (− 3)2 ;

b) 4 ⋅ (− 3)2 + 14 ;

v) 0 ⋅ (− 7 )2 + 32 ⋅ 0 ;

2

2

1  1 g)   −  −  .  3  4

33


6.

Re{i gi ravenkite: a) a 2 = 49 ;

7.

b) x 2 = 64 ;

v) a 2 = 1 ;

g) x 2 = 100.

Presmetaj ja vrednosta na kvadratniot koren: a)

9;

b) 144 ;

v)

6,25 ;

g)

169

.

225

8.

Presmetaj ja vrednosta na izrazot: a) 4 ⋅ 25 ;

9.

b) − 2 100 ;

v) − 10 0,01 ;

g)

625 . 5

v) 100 ⋅ 0,25 ;

g)

64 : 0,04 .

g)

36 x 2 : 81y 2 .

Odredi ja vrednosta na izrazot: 2

a)

62 ;

b)

 4   ; 7

10. Uprosti go izrazot: a)

34

64a 2 ;

b) 100x 2 ;

v)

25a 2 ; 64b 2


4. PRESMETUVAWE KVADRATEN KOREN OD RACIONALEN BROJ 5. POIM ZA IRACIONALEN BROJ. 6. REALNI BROEVI. 7. PRETSTAVUVAWE NA REALNITE BROEVI NA BROJNA OSKA. 1.

Presmetaj: a)

256 ;

b)

3600 ;

v) 12,25 ;

g)

27,04 .

2. So pomo{ na tablica ili digitron odredi ja pribli`nata vrednost na broevite na dve decimali: a) 28 ; b) 280 ; v) 12,5 ; g) 47,8 .

3.

Presmetaj ja vrednosta na izrazot: a)

4.

b) 2 ⋅ 25 − 3 ⋅ 25 − 3 2,25 − 0,6 2 .

So pomo{ na tablica ili digitron odredi ja vrednosta na: a)

5.

64 − 6,25 + 7,5 2 ;

2;

b)

3;

v) 10 ;

g)

200

so to~nost od 0,01.

Na brojnata oska pretstavi gi iracionalnite broevi: a) − 5 ;

b)

7;

v) − 6 ;

g) 13 .

35


6.

Re{i gi ravenkite: a) x 2 = 10 ;

7.

b) x 2 = 25 ;

v) a 2 = 50 ;

g) b 2 = 100 .

1   Dadeno e mno`estvoto: R = − 3,− 3 ,−2, , 3 ,2, 5  . Zapi{i gi mno`estvata tabelarno: 3   i A = {x x ∈ R i x ∈ Q} B = {x x ∈ R i x ∈ J}.

141 15 ; 2 ; ; a potoa sredi gi. 100 10

8.

Odredi gi decimalnite zapisi na broevite

9.

Na brojnata oska Ox , pretstavi gi realnite broevi: 1 a) ; b) 17 ; v) 3,75; 3

g) 10 .

10. Na brojnata oska na crte`ot pretstavi gi to~kite: A(-2), B(+4), C(-3,5), D( − 2 ), E( 2 ) i F( 1 + 2 )

36


Tema III

POLINOMI

1. ALGEBARSKI IZRAZ.BROJNA VREDNOST NA IZRAZ (BROJNI IZRAZI. IZRAZI SO PROMENLIVI) 1. Odredi ja brojnata vrednost na izrazot: (−4) 2 + 82 : 64 20 + 8 : 4 a) 23 − 4 ⋅ 2 ; b) ; v) ; 0,1 2 − 22

g) 2 4 ⋅ 32 − 5 ⋅ 6 2 + 8 : 4 ⋅ 2 .

2. Odredi koj od dadenite brojni izrazi nemaat (brojna vrednost) smisla i zo{to: 14 − 3 ⋅ 0,5 10 − 0,12 ⋅100 17 − 4 2 ⋅1 + 1 8 − 16 a) ; b) ; v) ; g) . 1 − 7 ⋅ 5 + 35 42 − 24 23 ⋅ 1 + 1 5 − 20 ⋅ 4

3. Odredi ja brojnata vrednost na algebarskiot izraz: 5a + b 2 a) 2 x 3 − 3x − 2 za x = −1 ; b) za a = −1 i b = −3 . 3b − a 2

4. Za racionalniot izraz A( x) =

x3 −1 , odredi A(−2) i A(0) . 3− x

43


5. Na polesen na~in opredeli ja vrednosta na izrazot: 3

 a 4 ⋅ a3   za a = −2 ; a)  5   a ⋅a 

2

 b 4 ⋅ b5   za b = −2 ; b)  3   b 

v)

x 4 ⋅ x3 ⋅ x 2 za x = −1 . x5 ⋅ x 4

− 2a 2 + a − 2 . Odredi ja negovata brojna vrednost za a ∈ (− 2,−1,0,1,24) . a2 −1 Za koi od dadenite vrednosti na promenlivata a izrazot ne e definiran?

6. Daden e izrazot P =

7. Odredi ja brojnata vrednost na izrazot: x 3 − 3x ⋅ y , za: a) x = 2, y = −1 ;

b) x = −2, y = 0 ;

v) x = 0, y = −1 ;

g) x =

1 y = −2. 3

8. Popolni ja tablicata na vrednosta na izrazite: ( a, b)

a−b

b−a

2a − b

2b − a

(1,2) (3,−1) (−2,−4) 1 ( ;−1,5) 2

[ to zabele`uva{? Kakvi se vrednosite na izrazite a − b i b − a t.e. 2a − b i 2b − a za ist par vrednosti na promenlivite a i b ?

44


2. POIM ZA MONOM. SLI^NI I SPROTIVNI MONOMI 1. Dovedi gi vo normalen vid monomite: 1 2 a) 2a ⋅ a 4 ; b) − 0,5ab 3 ⋅ (−6a 2 b 2 ) ; v) a 2 ⋅ (−2 a) ; 4 5

g)

2 1 x ⋅ 10 x 2 y ⋅ (− y 3 ) . 5 2

2. Odredi ja glavnata vrednost i koeficientot na monomot: 2 3 a) 5 x ⋅ (−2 y 4 ) ; b) − a 2b 3 ⋅ (− a) ; v) − 4axy 3 ; g) 2ab 2 ⋅ 3a 4 . 3 4

3. Napi{i monomi so: a) glavna vrednost xy 2 i koeficient −

1 ; 2

b) koeficient

1 a i glavna vrednost − x 3 y 2 ; 3

v) koeficient −1 i glavna vrednost − x 3 y ; g) glavna vrednost a 3b 2 i koeficient 1 .

4. Presmetaj ja brojnata vrednost na monomot: 1 1 a) − 4a 2b 3 za a = −1 i b = 2 ; b) − x 2 y za x = − i y = −4 . 2 2

45


5. Popolni ja tablicata: monom

sprotiven monom

sli~en monom

− 3axy 2 1 + bxy 3

− 0,5 x 2 y 4 1 4 axy 2

6. Odredi koi, od dadenive monomi se sli~ni: a) 2xy 3 ;

b) −3xy ;

v) −

1 xy ; 3

7. Odredi koi od slednive monomi se sprotivni: 1 1 a) − ab ; b) + 0,2a 2b 3 ; v) ab ; 3 3

46

g) 0,2 xy 3 .

g) −0,2ab .


3. BINOM. TRINOM. POLINOM. 4. STEPEN NA MONOMOT I POLINOMOT. 1. Od monomite: 4ax; − 2a 2 x, 7ay, − x 2 , −8, −3x, formuliraj eden a) binom;

b) trinom;

v) polinom.

2. Dovedi go polinomot vo normalen vid: a) 2 x ⋅ (3 y 2 ) − x 2 y − 4 x(−3 y ) ;

b) 1,5a 2 y 3 + 2,5ay 2 − 4,3ay 2 + 2,4a 2 y 3 .

3. Pretstavi gi vo vid na polinom broevite: a) xy ; b) yx ; v) xyz ;

g) abcd .

4. Odredi ja definicionata oblast na polinomot: 4a + 3b − 1 1 a) 2a 3 − 2a 2 + 3 ; b) x 2 y − x − 2 ; v) . 5 3

5. Svedi go polinomot vo normalen vid: a) 3a ⋅ ab ⋅ −a ⋅ 7b 2 + 4a ⋅ 3b 2 − 5a 2b ;

b)

1 1 3 3 3 3 3 3 2 x y − x y + a b − a 2b . 4 2 5 10

47


6. Odredi ja brojnata vrednost na polinomot: a) 2a 2 + 3a − 5 za a = −5 ;

7. Odredi go stepenot na monomot: 1 a) 2 xy 2 ; b) − a 2 ; 3

b) − 4 xy 2 − 3xy + x 2 + 2 y za x = −2 i y = −5

v) − 2 x 3 y 2 ;

1 g) 4 . 5

8. Zapi{i polinom {to e sprotiven na polinomot: a) 2 x 3 − 2 x 2 + x − 1 ;

b) − 0,2 xy 2 + 1,5 x 2 y 2 − 4,3x 3 y .

9. Odredi go stepenot na sekoj od dadenite polinomi so promenliva x i y a) − 3 − 2 y 3 + 4 y 5 + 6 xy 7 ;

b) − 0,5 x 6 + 4,6 x 3 y 5 − 2 xy 2 − 4 x .

10. Podredi gi spored stepenot na promenlivata x , po~nuvaj}i od najvisokiot stepen polinomite: b) 12 x 3 y 4 − 1,2 x 5 y + 0,5 x 7 . a) − 3x 2 − 4 x + 5 x 5 − 2 x 3 + 3 ;

48


5. SOBIRAWE I ODZEMAWE NA MONOMI 6. SPROTIVNI POLINOMI. OSLOBODUVAWE OD ZAGRADI 1. Odredi go zbirot na monomite: a) 2xy 2 , − xy 2 , − 3xy 2 , 0,5 xy 2 ;

b) − 4 x 2 y 3 , −2 xy , +

1 2 2 x y ,+2 xy . 2

2. Od monomot − 3x 3 y 2 odzemi go monomot: a) − 5 x 3 y 2 ;

b) + 3x 3 y 2 ;

v) − 3x 3 y 2 ;

g) x 3 y 2 .

3. Odredi go monomot x za koj e to~no ravenstvoto: 1 1 a) x − 3 a 3b 2 = a 3b 2 ; b) 4,5a 2 b − x = −3,2a 2 b . 2 4

4. Odredi momom {to e ednakov na izrazot: a) 4,5 xy 2 − 3xy 2 + 5,5 xy 2 − 2 xy 2 ;

b)

1 2 1 a b − a 2b + ba 2 . 4 3

5. Zapi{i polinom {to e sprotiven na polinomot: a) 2 x 3 − 2 x 2 + x − 1 ;

b) − 0,2 xy 2 + 1,5 x 2 y 2 − 4,3x 3 y .

49


6. Transformiraj go izrazot vo polimom od normalen vid: a) (2 − 2 x) + ( x 2 − 3x);

b) (a 3 − a − 4) − (−a + a 3 − 4) .

7. Uprosti go izrazot: a) ( x 3 + x 2 − x + 2) + (− x 3 + 2 x 2 − 2) ;

b) (2 x 2 − 4 x + 3) − (2 x 2 − 4 x + 3) .

8. Re{i gi ravenkite: a) 4 + 2a − (3a − 4) = 7 ;

b) (4 + 3x) − (7 x − 10) = 100 .

9. Doka`i deka vrednosta na izrazot: (3a − 2b + 1) − (5a + 3b − 2) − (2a − 5b − 1) ne zavisi od a i b .

10. Pretstavi go trinomot: a) 2a 2 − 4a + 2 ;

50

b) 3x 2 + 3x − 2 vo vid na zbir od dva binoma.


7. SOBIRAWE I ODZEMAWE NA POLINOMI 1. Transformiraj go vo normalen vid, polinomot: a) (2 x 2 + x − 1) + (− x 2 − x + 1) =

b) (4a 2b − 3ab 2 ) − (2a 2b − 3ab 2 ) =

2. Dadeni se polinomite: A = 2 x 2 − 3x + 1, B = x 2 + 2 x − 1 , i C = x 2 − 3 . Poka`i deka va`at zakonite: a) A + B = B + A ;

b) (A + B) + C = A + (B + C)

3. Odredi polinom A , koj e ednakov na zbirot na polinomite: 1 2 1 1 2 1 b) a) 2 x 3 y − 4 xy 2 + xy i x 3 y + 2 xy 2 − xy ; a b + ab + 5 . a b − ab − 3 i 2 4 4 3

4. Odredi polinom M , koj e ednakov na razlikata na polinomite: a) − 4 + x − 2 x 2 i 3x 2 − 2 x − 4 ;

b) ax 3 − 2 x 2 + x − 5 i − ax 3 + 4 x 2 − x + 5 .

51


5. Za polinomite: A = x 3 − 2 x 2 + 3x − 1 , B = −2 x 3 + 2 x 2 + x − 4 i C = 2 x 3 − x 2 + 2 x + 5 odredi: a) A + B + C ;

b) A + (B + C) ;

v) A − (B + C) ;

g) A − (B − C) .

6. Poka`i deka e to~no ravenstvoto: a) (3x − 2 x 2 + 4) + (2 x 2 − 3x − 4) = 0 ;

b) (−3a 3 x 2 + 2,4a 2 x 3 ) − (−3a 3 x 2 − 2,4a 2 x 3 ) = 4,8a 2 x 3 .

7. Za koja vrednost na x brojnata vrednost na izrazot (4 x + 5) − (− x − 5) e ednakva na 0.

8. Doka`i deka zbirot od koj bilo dvocifren broj i brojot napi{an so isti cifri, no po obraten red e deliv so 11.

52


8. MNO@EWE I DELEWE NA MONOMI 9. STEPENUVAWE NA MONOMI 10. MNO@EWE NA POLINOM SO MONOM 1. Presmetaj go proizvodot na monomite: a) a 4 ⋅ a 3 ;

2. Presmetaj: 3 a) − 4 x 4 y 2 ⋅ xy ; 4

b) 4ab 2 ⋅ (−3ab) ;

v) 2 x 2 ⋅ (−3x) ;

g) (−4 xy 2 ) ⋅ (−0,5 x 2 ) .

b) 6abx 2 ⋅ (−4ab 2 ) ;

v) (−0,6 x 2 y 3 ) ⋅ (−0,5 x 3 y 2 ) .

3. Odredi gi slednive proizvodi: a) 3x n y 2 ⋅ 3xy n ;

b)

1 n +1 2 4 n + 2 2 n x y ⋅ x y . 4 5

4. Izvr{i go nazna~enoto delewe na monomite: a) 24 x 5 y 4 c : (−6 xyc) ;

b) 3,6 x 4 y 2 : 0,12 x 4 y ;

1 1 v) − 1 a 4 x 4 y 2 : 1 a 2 x 2 y 2 . 3 2

5. Presmetaj: a) 0,4 x 7 y 5 z 4 : (−0,5 x 3 y 3 z 3 ) ;

2 1 b) − a 4 x 3 y 2 : (− a 3 xy 2 ) . 3 2

53


6. Stepenuvaj gi monomite: a) (2 x) 2 ;

b) (−3abx 3 ) 2 ;

v) (− xy 2 z ) 3 ;

1 g) (− a 2b 4 ) 3 . 2

7. Presmetaj: a) (4 xy ) 2 ;

b) −

3 2 2 a b ⋅ (−2ab 2 ) 3 ; 4

v) (4 x 2 y 3 ) 2 ⋅ (− xy 2 ) .

8. Presmetaj: a) (2 x − 3 y ) ⋅ 2 x ;

b) (6ab − 4ab 2 + 3a) ⋅ (−2ab) .

9. Odredi go polinomot P , taka {to: a) (a 2 + 1) ⋅ a + a 2 ⋅ (a + 1) = P + a 2 ;

b) P − (2 x 2 − x) = 3x ⋅ ( x 2 − 2 x − 1) .

10. Za koja vrednost na x brojnata vrednost na izrazot: a) 3( x + 2) + 6 e ednakva na 10;

54

b) 2( x − 1) + 3( x + 2) e ednakva na 8?


11. MNO@EWE NA POLINOMI 1. Presmetaj gi proizvodite: a) (2a − 1) ⋅ (a + 5) ;

b) ( x − 1) ⋅ (3x 2 − x + 3) ;

v) (4 xy + 2 x) ⋅ (3xy − 5 y ) .

2. Izvr{i gi nazna~enite operacii: a) ( x − 1) ⋅ ( x + 3) − ( x + 2) ⋅ ( x − 2) + (−4 x) ⋅ ( x + 2) ;

b) ( x 2 − 2 x + 1) ⋅ ( x + 1) − 4 x ⋅ ( x 2 − 3x − 1) .

3. Dadeni se polinomite: A = x + 1 , B = 3x + 2 , C = 2 x − 3 . Presmetaj: a) A ⋅ C ;

b) B ⋅ C ;

v) A ⋅ B ⋅ C ;

g) (C ⋅ A) ⋅ (−B) .

4. Pretstavi go, kako polinom vo normalen vid, izrazot: a) (6a 2 − 2a + 1)(3a − 2) − (2a 2 + 3) ⋅ ( x − 4) − 10 ;

b) (a 4 − a 3 + a 2 − a + 1) ⋅ (a + 1) .

55


5. Poka`i deka ravenstvoto e to~no: (a 2 − a + 1) ⋅ (a + 1) = (a 2 + a + 1)(a − 1) + 2 .

6. Odredi ja vrednosta na x za koja izrazot: 6 x 2 − (2 x − 3)(3x − 2) ima vrednost 7.

7. Doka`i deka za a ∈ N izrazot: a ⋅ (a + 5) − (a − 11)(a + 5) e deliv so 11.

8. Odredi A( x) − B( x) ⋅ C( x) ako A( x) + (2 x + 1) = x + 1 . B( x) − ( x + 1) = 2 x 2 − 1 i x 2 − C( x) = x + 2

9. Doka`i deka, pri sekoja vrednost na n, izrazot (n + 3) ⋅ (n − 2) + 5(n 2 − 2n + 1) + 3(3n + 2) e sekoga{ pozitiven.

56


12. FORMULI ZA SKRATENO MNO@EWE 1. Presmetaj gi proizvodite: a) (a − 2)(a + 2) ;

b) (2 x − 1)(2 x − 1) ;

v) (5 x − 3 y )(3 y + 5 x) .

2. Uprosti go izrazot: a) ( x − y )( x + y ) ⋅ ( x 2 + y 2 ),

b) (1 + a)(1 − a) ⋅ (1 + a 2 ) .

3. Transformiraj go izrazot vo vid na polinom vo normalen vid: a) ( x + 3) ⋅ ( x − 3) − x( x − 2) ;

b) ( x − 2 y )( x + 2 y ) − 2 x( x − y ) + 2 xy .

4. Presmetaj gi proizvodite: a) 23 ⋅17 ;

b) 7,5 ⋅ 6,5 ;

v) 98 ⋅ 102 ;

1 1 g) 10 ⋅ 9 . 2 2

5. So koristewe na formulata za kvadrat na binom, presmetaj: 1 a) ( x + 3) 2 ; b) (4 − a) 2 ; v) ( x + 1) 2 ; 2

g) (3a − 2b) 2 .

57


6. Odredi go monomot M , taka {to ravenstvoto da bide to~no: b) (3 + M ) 2 = 9 + 12 x 2 + 4 x 4 . a) (4a 2 − b) 2 = M − 8a 2b + b 2 ,

7. Uprosti go izrazot: a) (2 x − 5) 2 − (2 + 3x) 2 + (3 − x)(3 + x) ;

8. Re{i gi ravenkite: a) 2( x + 1) 2 − 2 x( x − 5) = 20 ;

b) (a − 1) 2 − 4(a + 1) 2 − 6(a + 1)(a − 1) .

b) (2 x + 1) 2 − 4( x − 1)( x + 1) = 22 .

9. So pomo{ na formulata za kvadrat na binom kvadriraj gi broevite: a) 512 ;

58

b) 199,52 ;

v) 1012 + 99 2 ;

g) 1052 .


13. DELEWE NA POLINOM SO MONOM. 14. DELEWE NA POLINOM SO POLINOM 1. Izvr{i gi nazna~enite operacii: a) (6 x + 18 y ) : 3 ;

b) (25 x 3 y 4 − 15 x 2 y 2 ) : (−5 xy ) ;

v) (0,4 x 3 − 1,2 x 2 + 2 x) : 0,8 x .

2. Transformiraj gi vo polinom vo normalen vid izrazite: 1 3 b) (16a 2 − 8a) : (−4a) − 4 ⋅ ( a − ) ⋅ 2 4

a) (10 x − 15) : 5 + 6 ⋅ (2 x − 4) ;

3. Odredi ja brojnata vrednost na izrazot: (169a 4b 3 − 78a 3b 4 + 13a 3b 3 ) : (−13a 3b 3 ) za a = −1 i b = 2 .

4. Re{i ja ravenkata: − 10 x 3 : (−2 x 2 ) − (2 x 2 − 3x) :

1 x = 9. 2

5. Odredi go koli~nikot: a) (a 2 + a − 12) : (a − 3) ;

b) ( x 2 − 2 x + 1) : ( x − 1) .

59


6. Prvo, podredi go polinomot, a potoa izvr{i go deleweto:

( 20 x − 41x

4

+ 16 x 3 + 20 x 7 − 10 ) : ( 5 x 4 − 4 ) .

7. Presmetaj gi koli~nicite: a) (a 2 − 1) : (a − 1) ;

b) (a 5 + 1) : (a + 1) ;

v) ( x 9 + 1) : ( x + 1) .

8. Re{i gi ravenkite: a) (2a 2 + 5a − 3) : (2a − 1) = 4 ;

b) (6 x 2 − 13x + 6) : (2 x − 3) = 4 .

9. Ako A( x) ⋅ B( x) = 8 x 3 − 12 x 2 y − 4 xy 2 + 2 y 3 i B( x) = 2 x + y . Opredeli go A( x) .

10. Izvr{i go nazna~enoto delewe na polinomot so ostatok: a) (15a 2 − 4a + 4) : (3a − 2) ;

60

b) ( x 4 + x 3 + 7 x 2 + 3x + 25) : (2 x 2 + 3x + 5) .


15. VIDOVI RACIONALNI IZRAZI 1. Odredi koi od slednive racionalni izrazi se celi, a koi drobni racionalni izrazi: a) 2 x ;

b)

2x − y ; 4

v)

2 ; y

g)

2a − 3 . a −1

2. Zapi{i dva algebarski izraza: a) koi se racionalni; b) koi ne se racionalni.

3. Odredi ja definicionata oblast na drobno-racionalniot izraz: 2 1 a2 − a −1 2x a) ; b) 2a − ; v) ; g) . y ( x − 1)( x + 2) x −1 1 − 3a

4. Skrati gi dropkite: x2 − 9 1 − x2 a) ; b) ; ( x − 3)( x + 3) ( x − 1) 2

v)

( a + b) 2 . a2 − b2

61


5. Za koja vrednost na a izrazot ne e definiran: a+b 3−a 4−a 1+ a ; b) ; v) ; g) ? a) (a + 3)(3a − 2) 1 − 3a 2 + 2a 1 + 5a

6. Odredi go mno`estvoto na dopu{tenite vrednosti na promenlivata, za koi racionalnite izrazi imaat smisla: a+b 2a 1 a) ; b) 2 ; v) . ( a + 3 )(3a − 2) 2a + 1 a −4

7. Za koja vrednost na promenlivata x dadeniot izraz ima smisla? 1− x 4− x 5+ x 2+ x a) ; b) ; v) ; g) . 2 − 3x x−6 2 + 2x 1 − 7x

62


16. RAZLO@UVAWE NA POLINOMITE NA PROSTI MNO@ITELI 1. Izvle~i gi zaedni~kite mno`iteli pred zagrada: a) 2a − 2 ;

b) 2 x − xy ;

v) 3ay − 6ax ;

g) −3x − 4 xa .

2. Razlo`i gi polinomite na mno`iteli: a) xy 3 − 4 xy + xy 2 ;

v) 6a 3b 4 − 12a 2b 2 + 24ab .

b) 10 x 2 y 3 − 15 xy + 20 x 3 y 4 ;

3. Skrati ja dropkata: ( x − 3) 2 2x + 2 y a) ; b) ; 27 − 9 x ( x + y) 2

v)

x( x − y ) + 2 xy ; ( x + y) ⋅ y

g)

2(a + b) + ab + a 2 . 3(a + b)

4. So razlo`uvawe na mno`iteli poka`i deka izrazot: a) 86 − 85 + 8 4 e deliv so 57;

b) 36 7 − 613 + 612 e deliv so 31.

5. Razlo`i go na prosti mno`iteli izrazot ax + bx + cx , a potoa presmetaj ja brojnata vrednost za: a = 41 , b = 34 , c = 25 i x = 0,55 .

6. Razlo`i gi slednive binomi na mno`iteli: a) 9a 2b 2 − 1 ; b) x 2 − 25 x 2 y 2 ;

v) 100 x 2 − 9 y 2 .

63


7. Presmetaj: a) 39 2 − 19 2 ;

3 2 v) (7 ) 2 − (5 ) 2 . 4 4

b) 1112 − 39 2 ;

8. Skrati gi dropkite: a 2 − 5a a) 2 ; a − 25

b)

4 − 2x ; x2 − 4

1 − x2 . x −1

v)

9. Doka`i deka, za koj bilo a ∈ N vrednosta na izrazot: a) (a + 11) 2 − a 2 e deliv so 11,

b) (4a + 7) 2 − 1 e deliv so 4.

10. Razlo`i gi na mno`iteli polinomite: a) 9 + 6 x + x 2 ;

b) x 2 − 4 xy + 4 y 2 ;

v)

1 2 + a + a2 ; 9 3

g) 16a 2 + 24ab + 9b 2 .

11. Presmetaj ja vrednosta na izrazot: a) x 2 + 6 x + 9 za x = −2 ;

b) 9a 2 − 12ab + 4b 2 za a =

1 1 i b=− . 3 2

12. Presmetaj ja na najednostaven na~in vrednosta na izrazot: a) 69 2 + 2 ⋅ 69 ⋅ 23 + 232 =

13. Skrati ja dropkata: 3xy − x a) ; 9y − 6y +1

64

b) 8,32 − 2 ⋅ 8,3 ⋅ 5,8 + 5,82 =

b)

9 x 2 − 15 x + 25 . 25 − 9 x 2


Tema IV

KRU@NICA I MNOGUAGOLNIK. PLO[ TINA 1. CENTRALEN AGOL. SVOJSTVA

1. Eden kru`en lak pretstavuva: 5 3 ; b) ; od kru`nicata. a) 12 5 Odredi go soodvetniot centralen agol, a potoa nacrtaj go vo dadenata kru`nica. a).

b).

2. Kolkav centralen agol odgovara na edna polukru`nica kako del od kru`nica? Odgovor: ________________________________ 3. Kolkav del od kru`nicata zafa}a eden centralen agol so golemina od 135O? Odgovor: ________________________________ 4. Centralniot agol na dadenata kru`nica razdeli go na tri agli, ~ii golemini se odnesuvaat kako 1 : 2 : 3.

5. Neka ABCDEF e {estagolnik ~ii temiwa le`at na edna kru`nica so centar vo to~ka O i pritoa AB = CD = EF i BC = DE = FA . Doka`i deka ∠AOC = 120O.

79


2. PERIFEREN AGOL. TALESOVA TEOREMA 1. Vo dadena kru`nica ozna~i periferen agol so golemina: b) 130O; a) 70O;

v) 90O.

2. Kolkav periferen agol odgovara na eden centralen agol so golemina: b) 43O; v) 235O 25’ 36�? a) 180O;

Odgovor: a) __________ ;

b) __________ ;

v) __________

3. Doka`i deka perifernite agli nad dva skladni kru`ni laka vo edna kru`nica se skladni.

4. Doka`i deka bisektrisite na site periferni agli nad ist kru`en lak se se~at vo sredinata S na toj lak.

5. Konstruiraj pravoagolen triagolnik, ako se poznati hipotenuzata c i visinata h c kon nea.

80


3. KONSTRUKCIJA NA TANGENTA NA KRU@NICA 1. Konstruiraj tangenta od to~kata A kon dadenata kru`nica k. a) b)

2. Kakvo mno`estvo od to~ki obrazuvaat centrite na kru`nicite koi se dopiraat: a) do kracite na daden konveksen agol, b) do dve paralelni pravi ?

Odgovor:

a) ____________________

b) ____________________

3. Neka pravite p i q se tangenti na kru`nica k pri {to p ≠ q i p || q. Doka`i deka dopirnite to~ki p ∩ k i q ∩ k se dijametralno sprotivni to~ki na kru`nicata.

4. Dadena e kru`nicata k i dve zaemnonormalni tangenti p i q na k. Kakvo mno`estvo od to~ki obrazuvaat presecite p ∩ q na site pravi p i q so toa svojstvo?

5. Neka ∠AOB e centralen agol na kru`nicata k. Ako a i b se tangenti na kru`nicata k vo to~kite A i B, kolkav e agolot me|u to~kite A i B?

81


4. TETIVEN ^ETIRIAGOLNIK 1. Dali okolu pravoagolen trapez mo`e da se opi{e kru`nica? Obrazlo`i go odgovorot i napravi crte`.

Odgovor: _________________________________ 2. Dali okolu eden ~etiriagolnik ~ii tri agli se: b) ∠A = 65O, ∠B = 115O, ∠D = 80O, a) ∠A = 50O, ∠B = 70O, ∠C = 130O, v) ∠B = 50O, ∠C = 60O, ∠D = 80O, mo`e da se opi{e kru`nica?

Odgovor:

a) __________ ;

b) __________ ;

v) __________.

3. Dali mo`e da se konstruira tetiven ~etiriagolnik, ako zbirot na tri negovi agli se ednakvi na: b) 130O, 150O, 40O, v) 20O, 70O, 160O? a) 80O, 56O, 124O,

Odgovor:

a) __________ ;

b) __________ ;

v) __________ .

4. Dali okolu deltoid mo`e da se opi{e kru`nica? Obrazlo`i go odgovorot?

Odgovor: _________________________________ 5. Vo dadenata kru`nica vpi{i ~etiriagolnik ~ii tri posledovatelni agli se 70O, 80O i 140O.

82


5. TANGENTEN ^ETIRIAGOLNIK 1. Vo koi paralelogrami mo`e da se vpi{e kru`nica? Zo{to?

Odgovor: ________________________________

2. Odredi ja dol`inata na ~etvrtata strana na eden ~etiriagolnik ABCD, za toj da bide tangenten, ako: b) BC = 6 cm, CD = 8 cm i DA = 5 cm . a) AB = 5 cm, CD = 7 cm i AD = 8 cm ;

Odgovor:

a) __________ ;

b) __________ ;

3. Dali postoi tangenten ~etiriagolnik ~ii tri posledovatelni strani imaat dol`ini 2 cm, 8 cm i 5 cm? Zo{to?

Odgovor: ________________________________ 4. Doka`i deka vo pravoagolnik {to ne e kvadrat ne mo`e da se vpi{e kru`nica!

5. Nacrtaj trapez vo koj mo`e da se vpi{e i se opi{e kru`nica.

83


6. OP[ TO ZA MNOGUAGOLNIKOT 1. Nacrtaj konveksen ~etiriagolnik i nekonveksen petagolnik.

2. Kolku dijagonali mo`e da se povle~at: a) od edno teme, b) od site temiwa na eden sedumagolnik?

Odgovor:

a) __________ ;

b) __________ .

3. Eden n – agolnik e takov {to vo nego mo`e da se povle~at vkupno n dijagonali. Koja e vrednosta na n?

Odgovor: _________________________________ 4. Odredi go {estiot agol na eden {estagolnik, ako pet negovi agli imaat golemini: 165O, 148O, 172O, 155O i 163O.

5. Dali postoi konveksen n – agolnik, ako osum negovi vnatre{ni agli se ednakvi na 130O? Zo{to? (Upatstvo: Razgledaj gi nadvore{nite agli!)

Odgovor: _________________________________

84


7. PRAVILNI MNOGUAGOLNICI 1. Nacrtaj mnoguagolnik so ednakvi: a) agli, b) strani, koj ne e pravilen mnoguagolnik.

2. Kaj koj pravilen mnoguagolnik eden nadvore{en agol iznesuva 45O?

3. Doka`i deka kaj sekoj pravilen mnoguagolnik, negoviot centralen agol e ednakov so nadvore{niot agol.

4. Neka za petagolnikot ABCDE, triagolnicite ABC, BCD, CDE, DEA i EAB se skladni ramnokraki triagolnici so temiwa vo B, C, D, E i A, soodvetno. Dali ABCDE e pravilen petagolnik?

Odgovor: ________________________________ 5. Ako A 1 A 2 A 3 A 4 ..... A n e pravilen n – agolnik, doka`i deka A 1 A 4 || A 2 A 3 .

85


8. OPI[ ANA I VPI[ ANA KRU@NICA 1. Odredi go mno`estvoto od site to~ki vo ramninata koi se ednakvo oddale~eni od: a) temiwata, b) sredinite na stranite, na eden pravilen mnoguagolnik.

Odgovor:

a) __________ ;

b) __________ .

2. Za koj pravilen n – agolnik karakteristi~niot triagolnik e ramnostran triagolnik?

Odgovor:

n = __________

3. Ako A 1 A 2 A 3 ..... A 2k e pravilen 2k – agolnik (k > 2), poka`i deka A 1 A 3 A 5 ..... A 2k-1 e pravilen k – agolnik.

4. Eden pravilen n – agolnik ima 9 oski na simetrija. Kolkav e negoviot centralen agol?

Odgovor: _________________________________ 5. Vo koj pravilen mnoguagolnik vnatre{niot agol e za 108O pogolem od centralniot agol?

Odgovor: _________________________________

86


9. KONSTRUKCIJA NA NEKOI PRAVILNI MNOGUAGOLNICI 1. Konstruiraj ramnostran (pravilen) triagolnik koj e vpi{an vo dadenata kru`nica.

2. Okolu dadenata kru`nica opi{i pravilen {estagolnik.

3. So pomo{ na aglomer, okolu dadenata kru`nica opi{i pravilen petagolnik.

4. So pomo{ na aglomer, konstruiraj pravilen petagolnik so strana a = 1,5 cm.

5. Vo dadenata kru`nica so pomo{ na aglomer vpi{i pravilen desetagolnik.

87


10. PITAGOROVA TEOREMA 1. Najdi ja dol`inata na ednata kateta, ako se dadeni hipotenuzata i drugata kateta: a) c = 20 cm, b = 16 cm ; b) c = 29 cm, b = 21 cm ; v) c = 7 cm, b = 3 cm .

Odgovor:

a) __________ ;

b) __________ ;

v) __________.

2. Strelkite na eden ~asovnik se dolgi 2,4 cm i 1,8 cm. Kolku se oddale~eni vrvovite na strelkite koga tie poka`uvaat 3 ~asot?

Odgovor: _________________________________ 3. Kolkav e dijametarot na opi{anata kru`nica okolu pravoagolen triagolnik, ~ii kateti se dolgi: a = 6 cm, b = 2,5 cm?

4. Katetite na eden pravoagolen triagolnik se dolgi 2,4 dm i 7 dm. Odredi ja dol`inata na te`i{nata linija {to e povle~ena kon hipotenuzata.

Odgovor: _________________________________ 5. Dali triagolnikot so strani: a) 15 cm, 2 dm i 2,5 dm ; b) 5 cm, 1 cm i 14 cm, e pravoagolen?

88


Odgovor:

a) __________ ;

b) __________ .

11. PRIMENA NA PITAGOROVA TEOREMA

1. Odredi ja katetata na ramnokrak pravoagolen triagolnik so hipotenuza c = 20 cm.

Odgovor: ________________________________ 2. Vo pravilen {estagolnik so strana 4 cm vpi{ana e kru`nica. Presmetaj go radiusot na taa kru`nica.

Odgovor: ________________________________ 3. Presmetaj go perimetarot na pravoagolen trapez, ako se poznati negovite osnovi a = 5,7 cm, b = 1,9 cm i visinata h = 2,5 cm.

Odgovor: ________________________________ 4. Centrite na dve ednakvi kru`nici so radius 3,9 cm se na rastojanie eden od drug 7,2 cm. Presmetajte ja dol`inata na nivnata zaedni~ka tetiva.

Odgovor: ________________________________ 5. Vo edna kru`nica so radius 5 cm e vpi{an ramnokrak triagolnik, kaj koj visinata kaj osnovata e dolga 6,4 cm. Odredete gi dol`inite na stranite na triagolnikot.

89


Odgovor: _________________________________

12. KONSTRUKCIJA NA TO^KI NA BROJNATA OSKA KOI ODGOVARAAT NA BROEVITE 2 , 3 , 5 , ...

1. Da se konstruira otse~ka so dol`ina: a) 10 cm; b) 13 cm.

2. Da se konstruira otse~ka so dol`ina: b) 21 cm. a) 15 cm;

3. Dadeni se otse~ki so dol`ini a i b. Da se konstruira otse~ka so dol`ina

a 2 + 9b 2 .

4. Dadeni se otse~ki so dol`ini a i b. Da se konstruira otse~ka so dol`ina

4b 2 − a 2 .

5. Dadeni se otse~ki so dol`ini a i b. Da se konstruira otse~ka so dol`ina

4ab .

(Upatstvo: 4ab = (a + b ) − (a − b ) ) 2

90

2


13. POIM ZA PLO[ TINA 1. Nacrtaj eden triagolnik i eden ~etiriaglonik koi se ednakvoplo{ni.

2. Plo{tinata od 4 m 2, izrazi ja vo: a) cm 2 ; b) mm 2 .

Odgovor:

a) __________ ;

b) __________ .

3. Daden e paralelogram ABCD. Na polupravata AB odredi to~ka M, taka {to triagolnikot AMB e ednakvoplo{ten so paralelogramot ABCD.

4. Dadeniot triagolnik ABC razdeli go na 4 ednakvoplo{ni delovi.

5. Neka F 1 i F 2 se dve geometriski figuri. Obidi se da doka`e{ deka: P (F 1 âˆŞ F 2 ) = P (F 1 ) + P (F 2 ) - P (F 1 ∊ F 2 ).

91


14. PLO[ TINA NA PRAVOAGOLNIK 1. Edna niva vo forma na pravoagolnik so strani 240 m i 180 m treba da se nasadi so lozje. Kolku lozi }e se nasadat na taa niva, ako rastojanieto na lozite vo redot e 1 m, a rastojanieto me|u redovite e 1,2 m?

Odgovor: _________________________________ 2. Kvadrat so strana 12 cm i pravoagolnik so edna strana 8 cm imaat ednakvi plo{tini. Koja od tie dve figuri ima pogolem perimetar?

Odgovor: _________________________________ 3. Konstruiraj kvadrat ~ija plo{tina e ednakva na zbirot od plo{tinite na dva dadeni kvadrata so strani a i b.

4. Vo kvadrat ABCD so strana 8 cm vpi{an e drug kvadrat MNKL, ~ii temiwa le`at na stranite na kvadratot ABCD. Presmetaj ja plo{tinata na kvadratot MNKL.

Odgovor: ________________________________ 5. Vo kru`nica so radius 6,5 cm vpi{an e pravoagolnik na koj ednata strana mu e dolga 5 cm. Presmetaj go perimetarot i plo{tinata na toj pravoagolnik.

Odgovor: _____________________________________________________________________________

92


15. PLO[ TINA NA PARALELOGRAM 1. Ako dol`inata na osnovata na paralelogramot ja zgolemime 3 pati, kako treba da ja promenime soodvetnata visina pri {to plo{tinata na paralelogramot da ostane nepromeneta?

Odgovor: ________________________________ 2. Romb so strana a = 8 cm ima plo{tina 43,2 cm 2. Presmetaj ja visinata na rombot.

Odgovor: ________________________________ 3. Edna niva ima forma na paralelogram so osnova 500 m i soodvetna visina 280 m. Za kolku dena taa }e bide izorana: a) so dva kowa koi za eden den izoruvaat 40 ari; b) od eden traktor koj izoruva po 3,5 ha dnevno?

Odgovor:

a) __________ ;

b) __________ .

4. Da se presmeta plo{tinata na eden romb ako se znae deka negovata strana ima dol`ina 6 cm, a pomaliot agol me|u stranite iznesuva 60O.

Odgovor: ________________________________ 5. Presekot na dijagonalite na eden paralelogram e na rastojanie 2,5 cm i 3 cm od pravite na koi le`at negovite strani. Presmetaj go perimetarot na toj paralelogram, ako negovata plo{tina iznesuva 60 cm 2.

Odgovor: ________________________________

93


16. PLO[ TINA NA TRIAGOLNIK 1. Stranata i soodvetnata visina na eden triagolnik se dolgi 5 cm i 8 cm. Mo`e li negovata plo{tina da bide ednakva na: a) 16 cm 2; b) 20 cm 2; v) 25 cm 2? Zo{to?

Odgovor:

a) __________ ;

b) __________ ;

v) __________ .

2. Presmetaj ja plo{tinata na ramnokrak triagolnik ~ij krak e b = 4,8 cm, a visinata {to Ă­ odgovara na osnovata ima dol`ina 3,6 cm.

Odgovor: _________________________________ 3. Vo kru`nica so radius 6,6 cm vpi{an e ramnostran triagolnik. Presmetaj gi perimetarot i plo{tinata na triagolnikot.

Odgovor: _________________________________ 4. Eden dvor vo forma na triagolnik ima strani 15 m, 10 m i 12 m. Presmetaj ja plo{tinata na dvorot.

Odgovor: _________________________________ 5. Poznato e deka triagolnikot ABC ima plo{tina P = 10 cm2 i perimetar 20 cm. Presmetaj go radiusot na vpi{anata kru`nica.

94


Odgovor: ________________________________

17. PLO[ TINA NA TRAPEZ

1. Da se presmeta plo{tinata na trapez ako negovite osnovi se 7 cm i 5 cm, a visinata mu e 5,5 cm.

Odgovor: ________________________________ 2. Osnovite na pravoagolen trapez se dolgi 2 dm i 2,5 dm, a podolgiot krak e 13 cm. Presmetajte go perimetarot i plo{tinata na toj trapez.

Odgovor: ________________________________ 3. Presmetajte ja plo{tinata na ramnokrak trapez, ako se poznati negovite osnovi: a = 26 cm, b = 12 cm i krakot c = 17 cm.

Odgovor: ________________________________ 4. Plo{tinata na eden ramnokrak trapez e P = 68 cm 2, a osnovite mu se dolgi 13 cm i 4 cm. Presmetaj go negoviot perimetar.

Odgovor: ________________________________ 5. Perimetarot na eden ramnokrak trapez iznesuva 149 cm, a osnovite mu se dolgi 63 cm i 32 cm. Presmetaj ja negovata plo{tina.

95


Odgovor: _________________________________

18. PLO[ TINA NA DELTOID

1. Dijagonalite na eden romb se dolgi d 1 = 7,3 dm i d 2 = 5,6 dm. Presmetaj ja negovata plo{tina.

Odgovor: _________________________________ 2. Plo{tinata na eden deltoid e 256 cm2. Presmetaj gi dol`inite na negovite dijagonali, ako se znae deka ednata dijagonala e dvapati podolga od drugata dijagonala.

Odgovor: _________________________________ 3. Presmetaj ja plo{tinata na eden tetiven deltoid, ako se znae deka podolgata strana ima dol`ina 3 cm, a pokratkata dijagonala isto taka ima dol`ina 3 cm.

Odgovor: _________________________________ 4. Presmetaj ja plo{tinata na eden deltoid, ako pokratkata negova strana ima dol`ina 2 cm i ako se znae deka dijagonalata koja ne e simetrala na deltoidot zafa}a agli od 45O i 60O so pomalata i pogolemata strana, soodvetno.

Odgovor: _________________________________ 5. Presmetaj ja plo{tinata na eden ~etiriagolnik so normalni dijagonali, ako negovite dijagonali imaat dol`ini d 1 = 7 cm i d 2 = 11 cm.

96


Odgovor: ________________________________

19. PLO[ TINA NA PRAVILEN MNOGUAGOLNIK

1. Radiusot na vpi{anata kru`nica vo ramnostran triagolnik e r = 2,5 cm. Presmetaj gi perimetarot i plo{tinata na triagolnikot.

Odgovor: ________________________________ 2. Stranata na pravilen {estagolnik e 3,4 cm. Presmetaj gi perimetarot i plo{tinata na {es tagolnikot.

Odgovor: ________________________________ 3. Stranata na eden pravilen osumagolnik e a = 2 cm. Da se presmeta plo{tinata na osumagolnikot, ako se znae deka radiusot na vpi{anata kru`nicata ima dol`ina r = 1 + 2 cm.

(

)

Odgovor: ________________________________ 4. Okolu kru`nica so radius r opi{ani se ramnostran triagolnik, kvadrat i pravilen {es tagolnik. Koj od opi{anite mnoguagolnici ima najgolema, a koj najmala plo{tina?

Odgovor: ________________________________ 5. Okolu kru`nica so radius 3 cm opi{an e petagolnik so perimetar 32 cm. Presmetaj ja plo{tinata na petagolnikot.

Odgovor: ________________________________

97


20. DOL@INA NA KRU@NICA 1. Presmetaj ja dol`inata na edna kru`nica so radius r = 3,14 cm.

Odgovor: _________________________________ 2. Okolu kvadrat so dijagonala d = 4,6 cm opi{ana e kru`nica. Presmetaj ja dol`inata na taa kru`nica.

Odgovor: _________________________________ 3. Trkalata na eden avtomobil imaat dijametar 0,6 m. Kolku zavrtuvawa }e napravi ednoto trkalo otkako avtomobilot }e izmine pat dolg 17 km?

Odgovor: _________________________________ 4. Edna trka~ka pateka ima radius 350 m. Eden motociklist taa kru`na pateka ja obikoluva 6 pati za 11 minuti. Presmetaj ja brzinata na motociklistot vo: a) metri vo sekunda, b) kilometri na ~as.

Odgovor: _________________________________ 5. Eden kru`en stolb mo`e da se namota to~no 4 pati so edno ja`e ~ija dol`ina e 5 m. Presmetaj go radiusot na stolbot.

98


Odgovor: ________________________________

21. DOL@INA NA KRU@EN LAK

1. Presmetaj ja dol`inata na kru`en lak od kru`nicata so radius r = 6,8 cm, {to odgovara na centralen agol od 75O.

Odgovor: ________________________________ 2. Eden kru`en obra~ so radius r = 0,7 m e prese~en i od nego e napraven kru`en lak {to odgovara na kru`nica so radius 0,9 m. Presmetaj go centralniot agol {to odgovara na toj kru`en lak.

Odgovor: ________________________________ 3. Pri vrteweto na Zemjata okolu svojata oska, kolkav pat izminuva sekoja to~ka od ekvatorot za vreme od: a) 1 ~as, b) 1 minuta, v) 1 sekunda? (Radiusot na Zemjata e 6370 km.)

Odgovor: ________________________________ 4. Ohrid i Belgrad se nao|aat pribli`no na ist meridijan. Presmetaj ja nivnata me|usebna oddale~enost, ako e poznato deka geografskata {irina na Ohrid e α 1 = 41O7’, a na Belgrad e α 2 = 44O48’. (Radiusot na Zemjata e 6370 km.)

Odgovor: ________________________________ 5. Eden gumen kai{ opfa}a tri kru`nici so radiusi 1 cm koi me|usebno se dopiraat (vidi go crte`ot). Presmetaj ja dol`inata na kai{ot.

99


Odgovor: _____________________________________

22. PLO[ TINA NA KRUG

1. Presmetaj ja plo{tinata na eden kru`en bazen, ako perimetarot na bazenot e 100 m.

Odgovor: _________________________________ 2. Kvadrat so strana 15,7 cm i eden krug imaat pribli`no ednakvi perimetri. Koja od tie dve figuri ima pogolema plo{tina i za kolku?

Odgovor: _________________________________ 3. Zemjotresot se {iri so brzina od 800 m/s. Presmetaj kolkava povr{ina mo`e da zafati zemjotresot po 5 sekundi od negoviot po~etok?

Odgovor: _________________________________ 4. Edna {uma ima forma na krug so radius 2 km. Drvjata vo {umata se rasporedeni pribli`no po 3 drva na sekoi 10 m2. Presmetaj go pribli`no brojot na drvata vo taa {uma.

Odgovor: _________________________________ 5. Edna kru`na sala so radius r = 10 m e poplo~ena so plo~ki vo forma na pravilen {estagolnik. Kolku plo~ki pribli`no se koristeni za poplo~uvawe, ako stranata na sekoja plo~ka iznesuva 10 cm ?

Odgovor: _________________________________

100


23. PLO[ TINA NA KRU@EN ISE^OK I KRU@EN PRSTEN 1. Plo{tina na eden krug e P = 140 cm 2. Presmetaj ja plo{tinata na kru`en ise~ok, {to mu odgovara na centralen agol od 70O.

Odgovor: ________________________________ 2. Perimetarot na eden krug e L = 25,2 cm, a dol`inata na lakot na eden kru`en ise~ok od nego e l = 8,4 cm. Odredi go centralniot agol i plo{tinata na kru`niot ise~ok.

Odgovor: ________________________________ 3. Dve koncentri~ni kru`nici imaat dol`ini L 1 = 9,42 dm i L 2 = 6,28 dm. Presmetaj ja plo{tinata i {irinata na kru`niot prsten, {to tie go obrazuvaat.

Odgovor: ________________________________ 4. ^etiri kru`nici so ednakvi radiusi r = 2,8 cm se dopiraat edna so druga odnadvor. Presmetaj ja plo{tinata na delot od ramninata me|u niv. Napravi crte`.

Odgovor: ________________________________ 5. Presmetaj go radiusot na kru`nicata, koja razdeluva daden krug so radius r na dve ednakvoplo{ni figuri – kru`en prsten i krug.

101


Odgovor: _________________________________

24. DIJAGRAMI

1. Eden zemjodelec posadil 3 dekari so zelen~uk, 2,6 dekari so ovo{je, a 3,5 dekari posadil so `itarici. Ovie podatoci pretstavi gi so: a) stolbest dijagram, b) sektoren dijagram.

2. Podatocite od sektorniot dijagram pretstavi gi so stolbest dijagram.

3. Vo edna fabrika vo poslednite 5 godini se proizvedeni slednite koli~estva na hrana: 1997 1600 toni

1998 1750 toni

1999 1870 toni

2000 1720 toni

2001 1690 toni

Ovie podatoci pretstavi gi so dijagram. Koj dijagram }e go izbere{?

4. Na eden natprevar na koj u~estvuvale 163 sportisti dodeleni se 5 zlatni, 16 srebreni i 40 bronzeni medali. Pretstavi gi ovie podatoci so sektoren dijagram. Na kolku delovi }e go podeli{ krugot?

5. Edna fabrika ima proizvedeno 105 proizvodi. 9% od proizvodite se so lo{ kvalitet, 53% se so zadovoluva~ki kvalitet, a ostanatite proizvodi se so prvoklasen kvalitet. Pretstavi gi ovie podatoci so sektoren i stolbest dijagram.

102


103


Tema V

FUNKCIJA. PROPORCIONALNOST.

1. DEKARTOV PROIZVOD NA MNO@ESTVA 1. Vo podredeniot par ( x, y ) , prva komponenta e _________ , a vtora komponenta e _________ .

2. Zapi{i gi site podredeni parovi koi mo`at da se formiraat od elementite na mno`estvoto A = {2,4,6}

Odgovor: ____________________________________________________________________________

3. Zapi{i gi site podredeni parovi ~ija prva komponenta mu pripa|a na mno`estvoto A = {2,5,6} , a vtora komponenta na mno`estvoto B = {a, b} .

Odgovor: ____________________________________________________________________________

4. Pretstavi gi so graf podredenite parovi: a) (4,5); b) (6,2); v) (3,3).

Odgovor:

5. Dadeni se mno`estvata A = {3,5,7} i B = {2,4} . Zapi{i go tabelarno i pretstavi go so graf mno`estvoto: a) A Ă— B = {

b)

115


6. Dadeni se mno`estvata A = {a, b} i B = {3,4,5}. Pretstavi go: tabelarno i so koordinantna {ema mno`estvoto B × A = Re{enie: a) B × A = {

b)

7. Dadeno e mno`estvoto A = {x x ∈ N i 4 < x ≤ 7} . Pretstavi go na a) tabelaren na~in i b) so graf mno`estvoto A × A = A 2 Re{enie: a) A 2 =

b)

8. Dadeno e mno`estvoto A × B = {(3,8), (5,4), (2,4), (3,4), (2,8), (5,9)} . Zapi{i gi tabelarno mno`estvata: A ={

B={

9. Dadeni se mno`estvata M = {1,3,5} i B = {2,4} . Formiraj gi mno`estvata A × B i B × A i obrazlo`i zo{to A × B ≠ B × A Re{enie: a) A × B = {

b) B × A = {

v) A × B ≠ B × A bidej}i ____________________________________________________________ 10. Dadeni se mno`estvata A = {0,1,2}, B = {2,3} i C = {4,5} . Poka`i deka e to~no slednovo ravenstvo: (A ∪ B) × C = (A × C) ∪ (B × C)

116


2. PRAVOAGOLEN KOORDINATEN SISTEM  3 1. Na brojnata oska odredi ja mestopolo`bata na to~kite: A(2,5) , B1  ; C(−3) ; 0(0) .  4 Odgovor:

2. Odredi go rastojanieto na to~kata M od pravite Ox i Oy na crte`ot.

4. Vo koj kvadrant se nao|aat to~kite M, T, N i K , crte` 2. Odgovor: M se nao|a vo ___________________ T se nao|a vo ____________________ N se nao|a vo ___________________ K se nao|a vo ___________________

5. Zapi{i gi koordinatite to~kite M, T, N i K , crte` 2.

na

Odgovor: Odgovor: To~kata M: od Ox e oddale~ena _____ edinici. od Oy e oddale~ena _____ edinici.

M( , ); T( , ); N( , ); K( , ). 6. Vo pravoagolniot koordinaten sistem pretstavi gi to~kite: A(−3,2) ; B(1,−4) ; C(3;3) ; D(0;−2) ; E(−1;0)

3. Odredi gi apscisata i ordinatata na 1 to~kata A(− ;2,5) 2 Odgovor: apscisa e: ___________________________ ordinata e: _________________________

117


7. Vo pravoagolen koordinaten sistem pretstavi ja otse~kata AB , ako: A(−2;−1) i B(3;4)

8. Vo pravoagolen koordinaten sistem pretstavi go triagolnikot ⌠ABC, ako A(1;−2) B(−1;2) i C(3,3)

118

9. Vo pravoagolen koordinaten sistem dadeni se koordinatite na to~kite A(−4;−2) i D(−2;3) . Konstruiraj go trapezot ABCD , na koj temiwata B i C se simetri~ni so A i D soodvetno vo odnos na koordinatnite oski.

10. Odredi gi koordinatite na temiwata na kvadrat na koj ednoto teme e to~kata A(−1;−2) , dve po dve strani mu se paralelni so koordinantnite oski i stranata mu e dolga 4 edinici.


3. RELACIJA 1. Ako A i B se neprazni mno`estva, sekoe podmno`estvo od Dekartoviot proizvod A × B se vika_______________________ od A vo B .

2. Vo mno`estvoto M so graf zadadena e relacijata R . Zapi{i ja tabelarno relacijata R .

Odgovor: R = ______________________________________________ 3. Vo mno`estvoto A = {1,2,3,4,5,6,7,8} zadadena e relacijata R “...... e za dva pogolemo od ..... “. Pretstavi ja relacijata : a) na tabelaren na~in;

Odgovor:

a) R = _______________________

b) na opisen na~in.

b) R = _______________________

4. Od mno`estvoto A = {2,4,6,8,10} kon mno`estvoto B = {1,2,3,4,5} dadena e relacijata: R ”..... e dva pati pomal od ..... “. a) Relacijata R pretstavi ja so graf; b) Grafikonot na relacijata R zapi{i go na tabelaren na~in. Odgovor: a)

b) R = _______________________ 5. Vo mno`estvoto M = {2,4,6} сe dadenи relaciите: а) R1 :"≤" и б) R2 :" =" . Пretstavi ги овие релации табеларно. Odgovor: a) b)

119


4. FUNKCIJA 1. Na crte`ot so graf dadeni se tri relacii. Opredeli: a) koja od tie relacii e funkcija; b) domenot i kodomenot na funkcijata.

Odgovor: a) Funkcija e _______ ; b) Domen e mno`estvo _________________________________ ; v) Kodomen e mno`estvo ___________________ . 2. Kaj funkcijata f : A → B , simbolot x ∈ A se vika ___________, a y ili f (x) ∈ B se vika ___________. 3. Od mno`estvoto A = {x x ∈ N i x ≤ 8} kon mno`estvoto B = {a, b, c} dadeno e preslikuvaweto f : ... ako x e prost broj, toga{ x → a , ako x e slo`en broj, toga{ x → b , ako x ne e ni prost ni slo`en broj, toga{ x → c . Preslikuvaweto treba da se pretstavi so graf.

4. Sekoja to~ka K( x, y ) od koordinatnata ramnina se preslikuva vo to~ka K ' ( y, x) . Pri toa preslikuvawe, vo koja to~ka }e se preslika sekoja od to~kite: A(−1,3); B(3,4); C(0,−2); D(−5,0) . Napravi crte`i:

Odgovor:

5. Funkcijata f e zadadena so grafikonot:

f = {(−3,0), (−2,1), (−1,2), (0,3), (1,4), (2,5)}. Opredeli gi : a) domenot na f ; b) kodomenot na f .

Odgovor: a) D = ____________________ ; 120

b) B = ____________________ ;


5. ZADAVAWE NA FUNKCII 1. Zadadeni se funkciite: f = {(1, a)(2, b)(3, c)(4, a )} q = {(1, a)(2, b)(3, c)} i h = {(1, b)(2, c)(3, a )(4, c)} . Koi od ovie funkcii se ednakvi: Odgovor: Ednakvi se funkciite _______________________________________________________ 2. Za edna funkcija f : A → B velime deka e zadadena, ako se poznati: __________________________________________________________________________________ 3. Traktor pri orawe pominuva prese~no po 4,5 km na ~as. Izrazi ja zavisnosta pome|u pominatiot pat i potro{enoto vreme: Odgovor:

4. Grafikot na funkcijata e f = {(−1,−1), (−2,0), (3,−1), (5,4), (2,1)} . Pretstavi ja funkcijata so tablica. Odgovor: x f(x)

5. Dnevnata temperatura vo edno mesto se meri sekoi 3 ~asa i e dobiena slednata tablica. ~as tem

6 -6

9 -2

12 8

15 6

18 2

21 0

24 -4

Od tablicata odredi: b) vo kolku ~asot temperaturata bila najvisoka a) f (9); f (18) f (24) . Odgovor:

a) f (9) = _______________; f (18) = _______________; f (24) = _______________; b) Temperaturata bila najvisoka vo _________ ~asot.

6. Grafikot na edna funkcija e f = {(−2,5), (−1,3)(1,−1), (2,−3), (0,1)} . Pretstavi ja funkcijata f (x) : a) tabelarno , b) grafi~ki. Re{enie: a)

b)

x f(x)

121


7. Zapi{i ja analiti~ki funkcijata definirana vo mno`estvoto Z so koja e izrazeno: “sekoj argument x e za tri pogolem od vrednosta na funkcijata”.

Odgovor: f (x) = ___________________________

8.

Pretstavi ja tabelarno i grafi~ki funkcijata f : A → Z f ( x) = x − 2 , ako definicionata oblast na funkcijata e mno`estvoto A = {− 3;−2;−1;0;1;2}

x f(x)

9. Dadena e funkcijata q : (A → Z) : x → x − 1 . Opredeli go mno`estvoto podredeni parovi ( x, y ) ∈ q , ako A = {− 5;−3;−1;0;2;4}

Odgovor: Γq = _____________________________

10.

Funkcijata h e zadadena so tablica. x

-2

-1

0

1

2

h(x)

5

3

1

-1

-3

Pretstavi ja grafi~ki.

122


6. RAZMER. PROPORCIJA. PRODOL@ENA PROPORCIJA 1. Koi od slednive koli~nici se razmeri 3 4 v) 9 m : 5 km ; a) 7 : 9; b) : ; 4 5

g) 0,7 : 2 kg ;

1 d) 0,1g : g . 2

Odgovor: Razmeri se: __________________________________________________________________

x . y Odgovor: Obratniot razmer e: _________________________________________________________

2. Zapi{i go obratniot razmer na razmerot

3. Napi{i gi to~no slednite odnosi: a) 64 km : 8 m;

v) 1,2m2:4dm2.

b) 5 ~asa : 2 min;

Odgovor: a) _____________

b)_______________

v)_________________

4. Slednive razmeri pretstavi gi so celi broevi: 2 1 a) 0,6 : 0,7; b) : ; 3 4 Re{enie:

3 v) 8 : 0,4 . 4

Odgovor: a) _____________

b)_______________

v)_________________

5. Presmetaj go nepoznatiot ~len na razmerot: 1 1 a) 72 : x = 6 b) x : 2 = 1 4 5

Odgovor: a) x = _____________ ;

b) x =_______________ . 123


6. Sostavi proporcija od slednive ravenstva na proizvodite: a) 20 ⋅ 30 = 50 ⋅ 12

b) 2 ⋅ 4 = 1,6 ⋅ 5

Odgovor: a) 20 : 50 =

b) 2 : 1,6 =

7. Kolku treba da iznesuva x vo ravenstvoto: 4,5 : x = 9 : 20 Odgovor: x = _____________ ; 8. Presmetaj go x vo slednata proporcija: 18 : ( x + 2) = 42 : 21

Odgovor: x = _____________ ; 9. Vo edno u~ili{te brojot na mомчиња sprema brojot na девојчиња se odnesuva kako 4 : 5. Opredeli kolku bile mомочиња, a kolku девојчиња, ako vo u~ili{teto imalo vkupno 990 u~enici. Re{enie:

Odgovor:

mомчиња bile _________________________ девојчиња ____________________________

10. Opredeli gi vnatre{nite agli na triagolnikot, ako nivnite golemini se odnesuvaat kako 3 : 4 : 5. Re{enie:

Odgovor:

124

α= ____________ ;

β= ____________ ;

ν= ____________ ;


7. PRAVA PROPORCIONALNOST 1. Veli~inite x i y se pravoproporcionalni so koeficient na proporcionalnosta

2 . 3

Zapi{i ja zavisnosta so formula. Odgovor:

2. Ako elementot x od mno`estvoto A (domen) se preslikuva vo elementot y od mno`estvoto B (kodomen) po formulata y = kx , toga{ zavisnosta me|u tie dve veli~ini e________ __________________________________________________________________________________ 3. Pravata proporcionalnost na veli~inite x i y se dadeni so podredenite parovi: (2,8), (3,12), (5,20). Odredi go koeficientot na proporcionalnosta:

k = ______ _________

Odgovor:

4. Eden avtobus se dvi`i so 75 km na ~as. Kolkav pat }e pomine za t ~asa.

Odgovor:

Za t ~asa }e pomine_______ km.

5. Veli~inite x i y se pravoproporcionalni so koeficient na proporcionalnost k = 4 . Opredeli go mno`estvoto na vrednosta y , ako x ∈ {− 3,−2,−1,0,4,5} .

Re{enie:

Odgovor: y ∈ { 6. Opredeli koi od veli~inite dadeni so tablicata se pravoproporcionalni. a)

b)

v)

x

3

6

9

12

x

4

8

12

16

x

9

12

15

18

y

5

10

15

20

y

6

12

8

4

y

3

4

5

6

Odgovor:

Pravoproporcionalni veli~ini se: __________________________________________ 125


7. Popolni ja tablicata, ako veli~inite x i y se pravoproporcionalni so koeficient na porcionalnosta k = 0,4 .

x

0.5

1,25

1

1 2

1 8

2

y

8. Popolni ja tablicata, ako e poznato deka veli~inite x i y se pravoproporcionalni: x

-3

-2 -10

y

5 10

8

25

50

9. Pretstavi ja grafi~ki pravata proporcionalnost y = −2 x x

-2

-1

0

1

2

3

y

1 10. Koeficientot na pravoproporciolanost e 2 . 2 Popolni ja tablicata i nacrtaj go nejziniot grafik

x y

126

-4

-2

0

2

4


8. OBRATNA PROPORCIONALNOST 1. Veli~inite x i y se obratnoproporcionalni. Zapi{i ja taa zavisnost so formula ako koeficientot na proporcionalnost e 5. Odgovor:

2. So koja od slednite formuli e pretstavena obratnata proporcionalnost. 2 1 3 a) y = − ; b) y = x ; v) y = 2 ⋅ x x 4 Odgovor:

Obratnoproporcionalni se: _________________.

3. Veli~inite x i y se obratnoproporcionalni so koeficient na proporcionalnost k = Odredi ja vrednosta na y , ako x = 0,6

1 . 2

Odgovor: y = _________ 4. Veli~inite x i y se obratnoproporcionalni so koeficient na proporcionalnosta

2 . 5

Odredi ja vrednosta x ako y = 0,4 .

Odgovor: x = _______ _____

5. Odnosot na dve proizvolni vrednosti od domenot e ednakov na obratniot odnos na : __________________________________________________________________________________

6. Veli~inite x i y se obratno proporcionalni so koeficient na proporcionalnost 4. Sostavi tablica za ovie veli~ini ako x ∈ {− 5,−2,−1,0,1,2,5}.

Re{enie:

x

-5

-2

-1

0

1

2

5

y

127


7. Ako x i y se obratnoproporcionalni veli~ini. Popolni ja tablicata otkako }e go odredi{ koeficientot na proporcionalnosta. Re{enie:

k = _____________________ 4

x

6

8

24

8

y

4

8. Veli~inite x i y zadadeni so tablicata se obratnoproporcionalni. Od tablicata opredeli go koeficientot na proporcionalnost i zapi{i ja formulata so koja e izrazena zavisnosta me|u x i y . x

24

12

y

2

1

6

3

1 2

1 4

a) k = ______

Odgovor:

b) y =

9. Popolni ja tablicata i nacrtaj go grafot na funkcijata y =

x

1

2

1 2

-

1 2

-1

-2

-4

y

10. Nacrtaj go grafikot na funkcijata y = −

x y

128

8 x

4

4 x


9. PROSTO TROJNO PRAVILO 1. Za {iewe na 12 ma{ki kostumi upotrebeno e 36 m {tof. Kolku metri e potrebno za {iewe na 16 takvi kostumi.

Odgovor:

Potrebni se _________ metri {tof.

2. Od 0,5 toni sve`i jabolka se dobivaat 95 kg suvi jabolka. Kolku kilogrami suvi jabolka }e se dobijat od 2,1 ton sve`i jabolka.

Odgovor:

] e se dobijat _________ jabolka.

3. Edna pumpa za voda dava 72 m3 voda za 4 ~asa i 12 minuti. Za kolku vreme pumpata }e dade 2143 m3 voda?

Odgovor:

Za vreme od ____________

4. 24 kravi se hranat so nekoja hrana 6 dena. Kolku dolgo }e se hranat so istata hrana 36 kravi?

Odgovor:

36 kravi }e se hranat _________ dena.

5. Eden avtomobil na 120 km vozewe tro{i 9 litri benzin. Kolku benzin }e potro{i na pat od 216 km?

Odgovor:

] e potro{i _________ litri benzin. 129


6. Edno ni{alo pravi 67 oscilacii (ni{awa) vo edna minuta. Za kolku sekundi }e napravi 2278 oscilacii?

Odgovor:

] e napravi za ________ oscilacii ___________ minuti.

7. Eden traktor so tri pluga mo`e da izora 84 dekari zemja. Kolku dekari zemja }e izora ako raboti so 4 pluga so ista brzina.

Odgovor:

] e izora ________ dekari zemja.

8. Eden zap~enik so 30 zapci pravi 80 zavrtuvawa vo minuta. Kolku zapci ima drug zap~enik koj e svrzan so nego, ako za isto vreme pravi 60 zavrtuvawa.

Odgovor:

Ima ________ zapci.

9. Edna rabota ja zapo~nale 33 rabotnici, i po planot bi ja zavr{ile za 80 dena. Me|utoa posle 16 dena rabotewe, 9 rabotnici se premesteni na drugo gradili{te. Za kolku dena e zavr{ena rabotata.

Odgovor:

Rabotata }e se zavr{i za ________ dena.

10. 32 rabotnika mo`at da asfaltiraat edna ulica za 12 dena. Brojot na rabotnicite se zgolemil za 16. Za kolku dena }e se zavr{i istata rabota.

Odgovor:

130

Istata rabota }e se zavr{i za _______ dena.


10. RABOTA SO PODATOCI 1. Prose~nata temperatura vo tekot na edna nedela vo mesec januari iznesuva: -9OC, -11,5 OC, -7,8 OC, -3,2 OC, 2 OC, 5,5 OC i 6 OC. Presmetaj ja srednata temperatura na vozduhot vo taa nedela: Re{enie:

Odgovor: Srednata temperatura e: _____ OC 2. Odredi ja geometriskata sredina na broevite 4 5 a) 3 i 27 b) i 36 5

Odgovor:

a) _______________ ;

b) _______________.

3. Vo nizata (podatoci) broevi: 1, 1, 2, 3, 4, 5, 5, 5, 6, 7 opredeli go modot.

Odgovor:

Mod _________

1 4. Dadena e nizata broevi 2; 2 ; 3; 3,5; 4; 5; 6. Opredeli go rangot. 3

Odgovor:

Rang _________

1 3 5. Opredeli go rangot, modot i medijanata na podatoci. 2; 3 ;6 ; 8; 8; 9; 9; 9; 12. 2 4

Odgovor:

a) Rang ___________ ;

b) Mod ___________ ;

v) Medijana ___________ 131


6. Vo edna kutija ima 15 beli, 25 crveni i 20 sini top~iwa. Koлку проценти од топчињата биле а) бели, б) црвени, в) сини?

Odgovor:

а) __________ ;

б) __________ ;

в) __________ .

7. Една коцка за играње фрли ја 100 пати и запиши ги добиените броеви. Потоа, врз основа на тие податоци, најди ја аритметичката средина, модот и медијаната.

Odgovor:

________________________________________________________________________

8. Од еден шпил на карти извлечи 20 карти по случаен избор, а потоа запиши ги добиените броеви од тој избор (за џандар запишувај број 12, за дама - број 13, а за поп - број 14). Од добиените податоци најди го рангот, аритметичката средина, модот и медијаната.

Odgovor:

________________________________________________________________________

9. Нека 0<a<b. Обиди се да докажeш дека за aритметичката средина А и геометриската средина G, важат неравенствата: a < A < G < b.

10. Најди низа од седум броеви, така што аритметичката средина да биде 5, медијаната да биде 5, модот да биде 6, а рангот да биде 10.

Odgovor: ___________________________________________ 132


TEST - 1 1. Dopolni gi re~enicite za da bide to~en iskazot a) Mno`estvoto P od site pravi {to se ___________________________________ so pravata a se vika ____________________________________ zadaden so pravata a. b) Dva vektora se sprotivni samo, ako _______________________________________________ 2.

Spored dadeniot crte` opredeli go vektorot: a) AC + CB

Odgovor:

a) AC + CB = ________________

3.

b) AD + DC + CB

b) AD + DC + CB = ________________ Spored dadeniot crte` opredeli go vektorot: a) AB − AD

Odgovor:

a) AB − AD =________________

b) AC − AB

b) AC − AB = ________________

  4. Vo paralelogramot ADPN vektorite AB = BC = CD = n i AM = MN = m .

  Izrazi go vektorot AP so pomo{ na vektorite m i n .

Odgovor:

AP = _________________________________________

5. Dopolni ja re~enicata za da bide to~en iskazot Pri translacija τ a pravata se preslikuva ___________________________________________ ___________________________________________________________________________________

6. Daden e trapezot ABCD (AB 7 CD), to~kite M i N sredini na kracite AD i BC, soodvetno i MP 7 BC. Izrazi go zbirot MN + MP so pomo{ na vektorite AB , DC i CB .

Odgovor:

MN + MP = ___________________________________

19


  7. Dadeni se nekolinearnite vektori m i n . Konstruiraj go vektorot:   a) m + n   b) m - n

Odgovor:   a) m + n = ________________   b) m - n = ________________

 8. Dadeni se pravite p, q i vektorot a . Konstruiraj otse~ka AB, taka {to A ∈ p, B ∈ q i  AB = a .

9. Nacrtaj kru`nica k(O,r=2,5cm), a potoa preslikaj ja so pomo{ na транслација за даден вектор AB , чија должина е 2,5cm.

10. Nacrtaj произволен триаголник ABC, а потоа транслтирај го за вектор АТ , каде што Т е тежиштето на дадениот триаголник.

20


TEST - 2 1. Dopolni ja re~enicata za da bide to~en iskazot a) Dve polupravi se istonaso~eni, ako le`at na edna prava i imaat _____________________ ______________________ili le`at na razli~ni paralelni pravi, ako ___________________ ___________________________________________________________________________________ b) Dva vektora se ednakvi, ako ______________________________________________________ 2. Spored dadeniot crte` opredeli go vektorot: a) AC + CB

Odgovor: 3.

a) AC + CB = ________________

b) DC + DA

b) DC + DA = ________________

Spored crte`ot opredeli go vektorot: a) AC − AB

Odgovor:

a) AC − AB = ________________

b) AD − AC

b) AD − AC = ________________

 Vo ∆ABC vektorite AM = MN = NB = a i  AP = PC = b . Izrazi go vektorot CB so   pomo{ na vektorite a i b .

4.

Odgovor:

CB = ___________________________________

5. Dopolni ja re~enicata za da bide to~en iskazot Pri translacijata τ a agolot se preslikuva vo ________________________________________ ___________________________________________________________________________________    6. Neka m , n i p se tri proizvolni vektori. Poka`i ja konstruktivno to~nosta na ravenst      voto (m + n ) + p = m + (n + p ) - asocijativniot zakon.

21


  7. Dadeni se kolinearnite vektori a i b . Konstruiraj go vektorot:   a) a + b   b) a − b

Odgovor:   a) a + b = ________________   b) a − b = ________________

8. Nacrtaj kru`nica k(O,r). So pomo{ na translacija τ m preslikaj ja kru`nicata, ako  m = 2 ⋅ OP i P to~ka od kru`nicata.

9. Nacrtaj две прави што се сечат, а потоа транслатирај ги за даден вектор AB .

10. Naцртај квадрат ABCD со центар во точка О. Транслатирај го квадратот за вектор CО .

22


TEST - 3 1. Dopolni ja re~enicata za da bide to~en iskazot a) Podredeniot par to~ki (A,B) go opredeluva ________________________________________   b) Vektorite m i n se kolinearni, ako ______________________________________________ 2.

Spored dadeniot crte` opredeli go vektorot b) BC + CD

a) AB + BD

Odgovor: 3.

a) AB + BD = ________________

b) BC + CD = ________________

Spored dadeniot crte` opredeli go vektorot a)

Odgovor:

4.

MN − MQ

a) MN − MQ = ________________

b) MN − QP

b) MN − QP = ________________

Daden e trapezot ABCD (AB||CD). Neka M i N se sredini na kracite AD i BC, soodvetno. Izrazi go vektorot MN so pomo{ na vektorite   AB = a i DC = b .

Odgovor:

MN = ___________________________________

5. Dopolni ja re~enicata za da bide to~en iskazot  Translacija τ za vektorot a se vika ____________________________pri koe na sekoja to~ka M i se pridru`uva ___________________________________________________________ 6.

Otse~kite AA 1 , BB 1 i CC 1 se тежишни линии na ∆ABC. Izrazi go vektorot AA1 + BB1 + CC1 so    pomo{ na vektorite AB = c , BC = a , CA = b .

Odgovor:

AA1 + BB1 + CC1 = _______________________

23


  7. Dadeni se nekolinearnite vektori a i b . Opredeli go konstruktivno vektorot:   a) a + b   b) a − b

Odgovor:   a) a + b = ________________   b) a − b = ________________ 8.

Preslikaj go ramnokrakiot ∆ABC so pomo{ na  translacijata τ a , kade vektorot a = AM i M e sredi{na to~ka na stranata BC od ∆ABC.

9.

Daden e pravoagolnikot ABCD i to~kata O kako presek na dijagonalite AC i BD. Izvr{i транслација на правоаголникот за вектор OB .

10. Nad stranite AB i CD od paralelogramot ABCD konstruirani se kvadratite ABB 1 A 1 i DCC 1 D 1 , taka {to vektorite AA1 i DD1 se istonaso~ni. Ako prese~nite to~ki od dijagonalite na tie kvadrati se M i N, toga{ doka`i deka MN = BC . Dadeno:

ABB 1 A 1 i DCC 1 D 1 M=AB 1 ∩BA 1 ; N=DC 1 ∩CD 1 Tvrdime: MN = BC

Dokaz:

24


TEST - 4 1. Dopolni gi re~enicite za da bide to~en iskazot a) Vektorot e napolno opredelen so _________________________________________________ b) Niz sekoja to~ka od ramninata _______________________________________ daden pravec. 2.

Spored dadeniot crte` opredeli go vektorot a) AB + BC

Odgovor: 3.

a) AB + BC = ________________

b) AP + MN

b) AP + MN = ________________

Spored dadeniot crte` opredeli go vektorot a) AB − AD

Odgovor: 4.

a) AB − AD = ________________

b). AB − DC

b) AB − DC = ________________

Dadena e otse~kata MN, to~kata P sredina na MN i to~kata S ∉ AB. Izrazi go vektorot SP so pomo{ na vektorot SM i SN .

Odgovor:

SP = ___________________________________

5. Dopolni gi re~enicite za da bide to~en iskazot Translacijata τ a gi ima slednite svojstva: - __________________________________________________________________________________ - __________________________________________________________________________________ - __________________________________________________________________________________

6.

Vo ∆ABC, to~kite M, N i P se sredini na stranite AB, BC i AC, soodvetno. Izrazi go vektorot PN + PM so vektorite AB i CB .

Odgovor:

PN + PM = _____________________________

25


  7. Dadeni se nekolinearnite vektori a i b . Konstruiraj go vektorot:   a) a + b   b) a − b

Odgovor:   a) a + b = ________________   b) a − b = ________________

So pomo{ na translacija τ a preslikaj go  rombot ABCD za vektorot a = AB − AD .

8.

9. Konstruiraj ramnostran triagolnik ∆ABC so strana 3 cm. Потоа тој триаголник транслатирај го така што темето С да дојде во центарот О на рамностраниот триаголник.

10. So pomo{ na translacija konstruiraj trapez ABCD, ako se dadeni site strani, t.e. AB = a , BC = d , CD = b i AD = c Dadeno

26

Skica

Konstrukcija


TEST - 5 1. Dopolni gi re~enicite za da bide to~en iskazot a) Za sekoi dva vektora va`i ____________________________ zakon, a za sekoi tri vektori va`i ____________________________ zakon.  b) Proizvod na nenulti vektor a so brojot K ≠ 0 se vika ______________________________ 2.

Spored dadeniot crte` opredeli go vektorot a) AB + BC

Odgovor: 3.

a) AB + BC = ________________

b) AB + BC + CD

b) AB + BC + CD = ________________

Spored dadeniot crte` opredeli go vektorot a) AC − AB

Odgovor:

a) AC − AB = ________________

b) AD − AC

b) AD − AC = ________________

4. Po te~enieto na edna reka se dvi`i brod so brzina 42 km/h, a sproti te~enieto na rekata so brzina 28 km/h. Kolkava e brzinata samo na brodot, a kolkava na vodata na rekata.

Odgovor: Brzinata na brodot e _______________________, a na vodata e _____________________ 5. Dopolni gi re~enicite za da bide to~en iskazot  a) Translacijata τ za vektorot - a , t.e. τ −a e _______________________ na translacija τ a . b) Скаларни величини се: ____________________________________________________________ Векторски величини се: ____________________________________________________________

6. Neka vektorot AB = CD i AS = CT . Doka`i deka SB = TD . Dadeno:

AB = CD i AS = CT

Tvrdime: SB = TD Dokaz:

27


   7. Dadeni se nekolinearnite vektori a , b i c . Odredi go konstruktivno vektorot:    a) a + b + c   b) a − c

Odgovor:    a) a + b + c = ______________________   b) a − c = ______________________

8. Dadeni se dva agla so zaemnoparalelni kraci od koi edniot par kraci se isto naso~eni, a drugiot sprotivno naso~eni. Doka`i so pomo{ na translacija deka tie agli se suplementni. Dadeno: OA↑↓O 1 A 1 i OB↑↑O 1 B 1 Tvrdime: ∠AOB + ∠A 1 O 1 B 1 = 180o Dokaz:

9.

Daden e trapezot ABCD (AB||CD). Izvr{i trans lacija na trapezot za vektorot x = AD + DC .

10. Dadeni se dve skladni kru`nici k 1 (S 1 ,r) i k 2 (S 2 ,r). Odredi go векторот на транслација a таков што τ a (k 2 ) = k 1 .

Odgovor: Векторот на транслација е _________.

28


TEST - 1 1. Izvr{i gi nazna~enite operacii so sistemi: a ⋅ a3 ⋅ a0 ⋅ a5 =

a)

b) (a 3 ) 7 = v)

3x 4 = 6x 2

g) (

x3 ⋅ x ⋅ x8 2 ) = x ⋅ x3 ⋅ x2

2. Presmetaj i izvr{i proverka: a) 3600 =

b) 12,25 =

3. Presmetaj ja vrednosta na izrazot: 2 2 ⋅ 4,5 − 5 2 ⋅ (−0,8) 2 =

4. Докажи дека

5 е ирационален број.

5. Нацртај бројна права и нанеси ги броевите: 2,6; -1,4;

2 +1;

3 −1 ; −

2 ; 2,5, 3

a potoa zapi{i gi ovie broevi po golemina, po~nuvaj}i od najmaliot.

37


TEST - 2 1. Zapi{i gi kako stepen so osnova x, slednive izrazi: a) x 3 ⋅ x ⋅ x 2 = b) (x 6 ) 4 = x12 = x3 x5 ⋅ x ⋅ x 4 g) ( 2 3 ) 2 = x ⋅x

v)

2. Presmetaj со помош на таблица i izvr{i proverka: b) 27,04 = a) 2116 =

3. Presmetaj ja vrednosta na izrazot: 3 1 ( ) 2 ⋅ (−2) 3 − ( ) 3 ⋅ 4 2 = 4 2

4. Dali ima pove}e santimetri vo eden kilometer ili ima pove}e kubni santimetri vo eden kuben metar? Re{enie:

5. Zaokru`i gi to~nite iskazi: a) Sekoj priroden broj e cel i racionalen, no ne e iracionelen broj. b) Sekoj cel broj e racionalen, no ne e realen broj. v) Postoi realen broj koj e racionelen i iracionalen broj. g) Ne postoi iracionalen broj koj e i cel broj. d) Sekoj realen broj e racionalen ili iracionalen broj, no ne mo`e da bide i racionalen i iracionalen broj. |) Zbirot na dva iracionalni broja e sekoga{ iracionalen broj. e) Proizvodot na dva iracionalni broja e sekoga{ racionalen broj. 38


TEST - 3 1. Izvr{i gi nazna~enite operacii so stepeni: a) x 6 ⋅ x 5 ⋅ x 0 = b) x10 : x15 = v) (x 3 ) 5 = g)

x 7 ⋅ ( x 2 )3 = x6 ⋅ (x3 )2

2. Presmetaj i izvr{i proverka: a) 57 ;

b)

38,5 .

3. Presmetaj ja vrednosta na izrazot: 39 2 + 4 ⋅ (29 ⋅ 5 + 7 2 ) =

4. Uprosti go izrazot ili iracionalen broj.

5 5 ⋅ 5 7 , a potoa odgovori dali toj izraz pretstavuva racionalen

Re{enie:

5. Daden e izrazot A ( a ) = a) A ( −2) ,

a 2 − 3a + 1 . Оdredi: a −1

b) A ( 2)

39


TEST - 4 1. Zapi{i gi kako stepen so osnova a izrazite: a) a 5 ⋅ a 3 ⋅ a 0 ⋅ a = b) (a 4 ) 8 = a ⋅ a2 ⋅ a3 3 ) = a3 ⋅ a4 ⋅ a5 a 20 g) 10 = a

v) (

2. Presmetaj (со дигитрон или таблица) i izvr{i proverka: b) 2,25 = a) 13600 =

3. Presmetaj ja vrednosta na izrazot: 2 3 + (−3) 3 − (−2) 3 + (−1) 5 =

4. Daden e izrazot C ( y ) =

1− 4y − y3 y2 −1

. Odredi:

a) C ( −2) ,

5.

b) C

1 ( ) 2

Odговори на следните прашања:

а) Колку грами има во еден тон? б) Колку нанометри има во еден милиметар? Одговор: а)

40

б)


TEST - 5 1. Presmetaj: a) x 5 ⋅ x 8 ⋅ x 0 = b) (2 x 3 ⋅ y 4 )2 = v) x14 : x 7 = g) (

2x3 ⋅ y 4 2 ) = x⋅ y

2. Presmetaj i izvr{i proverka: a) 625 =

b)

94,437 =

3. Presmetaj ja vrednosta na izrazot: 5 − (−2) 3 ⋅ 3 + 32 : 9 − 4 ⋅12 : 2 3 =

4. Daden e izrazot A ( x ) = a) A ( −4) ;

5. Докажи дека Решение:

2x 2 − 2x − 2 . Odredi: x−2 b) A ( 0) .

3 + 1 е ирационален број.

41


TEST - 6 1. Presmetaj ja vrednosta na izrazot: a) (−2) 3 = b) 10 6 = v) (−1)100 = g) (0,1) 4 =

2. Presmetaj i izvr{i proverka: a) 961 =

3. Daden e izrazot B(x ) = a) B(0) ;

2x − x 2 + 3 x2 +1

b) 42,25 =

. Odredi: b) B(− 1) .

4. Presmetaj ja vrednosta na изразот: a) (−1) 200 = b) (0,1) 4 =

5. Daden e izrazot C ( x ) = a) C

42

1 ( ) 2

;

x2 − x + 5 . Odredi: x 2 −1

b) C (−2) .


TEST - 1 1. Od monomite: xy , − 2 x 2 y 2 , −3xy i

1 2 2 x y 2

a) sostavi polinom b) svedi go vo normalen vid v) odredi go stepenot na polinomot po (x) г) одреди го спротивниот полином 2. Za polinomite: A = 2 x 3 − x 2 + x − 1 , B = − x 2 + 2 x − 4 i C = x 3 + 4 x 2 − x − 2 , odredi C − (A + B) .

3. Izvr{i gi nazna~enite operacii: 3 5 4 2 x y z : (− x 2 y 3 ) = 4 3 2 3 3 b) (−6 x y ) =

a)

v) (4 x 3 − 2ax 2 ) ⋅ (−3ax 2 ) = g) (6 xy 2 − 3xy + 2)(−2 x 2 y 2 + 3xy ) = 1 3 4 x y . Оdredi: 2 a) sli~en monom -

4. Daden e monomot

b) sprotiven monom v) stepen na monomot g) koeficient na monomot -

5. Zbirot na polinomite: 4 x 2 − 3x + 2 i 5 − 4 x 2 − 4 x namali go za trinomot x 2 − 6 x + 4 .

65


6. Izvr{i gi nazna~enite operacii: 1 2 3 2 2 x y ⋅ xy = 2 3 1 2 4 3 b) (− a b ) = 2 2 v) a (2a 3 − a 2 − 2a) =

a) −

g) (5 x 2 − 2 x + 1)(2 x − 2) =

7. Pretstavi go so polinom vo normalen vid, proizvodot: a) ( x − 3)( x + 3) = b) (x − 2) 2 = v) 72 2 =

8. Izvr{i gi nazna~enite operacii: 3 7 2 a) ( ab 2 − a 2 b 3 ) : ab ; 4 8 3

b) (9 x 5 − 13x 2 + 12 + 6 x 7 − 7 x 4 ) : (−7 x 2 + 4 + 3x 5 ) .

9. Razlo`i go polinomot na prosti mno`iteli: b) 4ab + 12ac − 8ad ; a) a 3 + 4a 2 + 4a + 16 ;

10. Doka`i go identitetot: (10n + 5) 2 = 100n(n + 1) + 25 .

66

v) b( x − 3) + c( x − 3) + 3 − x .


TEST - 2 1. Pretstavi go kako polinom vo normalen vid, proizvodot: a) (5 − 3a) 2 = b) 7,5 ⋅ 6,5 = v) (4 x 2 + 3) 2 =

2. Izvr{i gi nazna~enite operacii: 1 a) (a + b − 1) : = 2

b) (a 2 + 5ab + 6b 2 ) : (a + 2b) =

3. Razlo`i go polinomot na prosti mno`iteli: a) 9a (2 y − 3) + 7b(3 − 2 y ) = b) 6 xy 2 − 18 x 2 y + 12 x 2 y 2 = v) xa + yb + xb + ya =

4. Daden e monomot − 0,5a 3 xy 2 . Оdredi: a) sli~en monom b) sprotiven monom v) stepen na monomot g) koeficient na monomot -

5. Od polinomot 2a 3 − 4a 2 + 6a − 3 odzemi go zbirot na polinomite: − a 3 + 4a − 2a 2 + 5 i a 3 − 2a 2 + 3a − 1 .

67


6. Izvr{i gi nazna~enite operacii: a) 0,1ax ⋅ 12ax 2 y = b) (−0,2 x 2 y 3 ) 2 = v) 2 x 2 y (2 x 2 − 2 x − y ) = g) (a 2 + 2a − 7) ⋅ (a − 1) = 7. Pretstavi go kako polinom vo normalen vid proizvodot: a) (a + 5)(a − 5) = b) (a − 3) 2 = v) (2a + 3) 2 = 8. Izvr{i gi nazna~enite operacii: a) (16 x 3 y − 18 x 4 y 2 ) : 4 x =

b) (−8 + 22 x − 12 x 2 − 12 x 3 + 9 x 4 ) : (3x − 4) =

9. Razlo`i go polinomot na prosti mno`iteli: a) 9a 3b 2 − 6a 2 b + 12a 2 b 3 ; b) n( x − y ) + nx − ny ;

10. Doka`i go identitetot: n(n + 1)(2n + 1) (n − 1)(2n − 1) ⋅ n − = n2 . 6 6

68

v) a 2 x + a 2 y − ax − ay + x + y .


TEST - 3 1. Od monomite: − a b ; −3ab ; 2 3

1 ab i − 7 a 2 b 3 2

a) sostavi polinom b) svedi go vo normalen vid v) odredi go stepenot na polinomot po (a, b) g) odredi go sprotivniot polinom -

2. Odredi go polinomot M , taka {to: (2 x 2 − x + 3x 3 + 1) − M = 2 x 3 − 2 x 2 − x − 4 .

3. Doka`i deka za sekoja vrednost na x izrazot: (3x − 4)(7 x + 8) − 1,5 x ⋅ (24 x + 4) − 5(1 − 2 x) e negativen.

3 4. Daden e monomot − a 2 b 4 . Оdredi: 4 a) koeficient na monomot -

b) stepen na monomot v) sli~en monom g) sprotiven monom -

5. Na polinomot − 3x 3 − 5 x 2 + 6 x − 3 , dodaj ja razlikata na polinomite: 2 x 2 − 5 x + 4 x 3 − 5 i 1 − x 3 + 2 x 2 − 3x .

69


6. Izvr{i gi nazna~enite operacii: a) (2 xy ) ⋅ (−3xy 2 ) = b) ( x 2 y 3 ) n = v) 4 ⋅ (2a − 3b + c) = g) ( x − 2 y ) ⋅ ( x 2 − xy − y 2 ) = 7. Pretstavi go vo normalen vid proizvodot: a) (2 x + 5) 2 = b) (2 x − 7)(2 x + 7) = v) 692 = 8. Izvr{i gi nazna~enite operacii: a) (25ax 2 y − 15 x 2 y 3b) : (−5ab) ;

b) (9 x 2 − 15 x − 10 x 3 + 6 x 4 ) : (2 x 2 + 3) .

9. Razlo`i go polinomot na prosti mno`iteli: a) − m 3 x − 6m 2 x + 3mx ; b) p ( x + y + 1) − q ( x + y + 1) + r ( x + y + 1) ;

10. Doka`i go identitetot: 1 1 (a + ) 2 = a(a + 1) + . 2 4

70

v) az 2 + bz 2 + az + bz + a + b .


TEST - 4 1. Izvr{i gi nazna~enite operacii: a) − 7 x 3 y 2 z ⋅ (−5 x 2 y 2 ) = 3 3 2 xy c) = 4 v) (4a 3 − 2ax 2 ) ⋅ 3ax 3 =

b) (−

g) (12 x 3 + x 2 − 9) ⋅ (4 x + 3) =

2. Pretstavi go kako polinomot vo normalen vid, proizvodot: a) (2a − 3b) 2 = 1 3 b) ( x + y ) 2 = 2 4

v) 98 ⋅102 =

3. Izvr{i gi nazna~enite operacii: a) (2,4 x 4 − 0,04 x 3 + 1,32 x 2 ) : 0,4 x 2 =

b) (3x 3 − 5 x 2 + 9 x − 15) : (3x − 5) =

4. Dovedi go vo normalen vid monomot − x 2 y 3 ⋅ (−2 x 3 ) ⋅ (− y 3 ) , a potoa odredi: a) sprotiven monom b) sli~en monom v) stepen na monomot g) koeficient na monomot -

5. Od zbirot na polinomite 4ab − b 2 + 2a 2 i 2a 2 − 5ab + 5b 2 , odzemi ja razlikata na polinomite 2ab + 3a 2 − 4b 2 i − 3b 2 + 2ab + 3a 2 .

71


6. Izvr{i gi nazna~enite operacii: 1 2 a) 4 x 2 y 2 ⋅ xy : (− x 3 y ) = 4 3 2 b) (−0,5 xy ) ⋅ (4 xy 2 ) 3 =

v) − 4 x(−2ax + 3ax 2 − x 3 ) = g) ( x 2 − x + 1) ⋅ ( x 2 − 4 xy + 3 y 2 ) =

7. Izvr{i gi nazna~enite operacii: a) (2a − 3) 2 = b) (3x − y )(3x + y ) = v) 732 =

8. Pretstavi go kako polinom vo normalen vid proizvodot: a) (−4,8a 3b 5 + 6a 2 b 4 − 2ab 3 ) : 2ab 3 = b) (a 7 − 1) : (a − 1) =

9. Razlo`i go polinomot na prosti mno`iteli: b) a ( x + 1) − b( x + 1) ; a) 6 xy 4 + 12 x 2 y 3 − 3xy 2 ;

10. Ako b + c = 10 , toga{ (10a + b)(10a + c) = 100a(a + 1) + bc . Doka`i!

72

v) 4a 2 + 2ab + 2ac + bc .


TEST - 5 1. Razlo`i go polinomot na prosti mno`iteli: a)

5 3 4 1 1 m n + 2 m4n2 − 3 m2n2 = 9 3 3

b) 4 x 2 y (a + 2) − 8 xy 2 (a + 2) = v) a 2 x − b 2 y − a 2 y + b 2 x =

2.

Od monomite:

1 1 2 ac , −4ac , − ac 2 i 10ac 2 2

a) sostavi polinom b) svedi go vo normalen vid v) odredi go stepenot na polinomot po (a, c) g) odredi go sprotivniot polinom 3. Doka`i deka ravenstvoto e to~no: (4m + n) 2 − 2(m + n)(m − n) − 8mn = 14m 2 + 3n 2 .

4. Dovedi go vo normalen vid monomot

1 2 x y ⋅ 9 x(−2 xy 2 ) , a potoa odredi: 3

a) koeficient na monomot b) stepen na monomot v) sprotiven monom g) sli~en monom -

5. Presmetaj A + (B − C ) , ako e: A = 2 x 3 − 6 x 2 + 5 x − 2 ; B = − x 3 − 4 x 2 + 3x − 6 i C = −2 x 3 + 2 x 2 + 4 x − 1 .

73


6. Izvr{i gi nazna~enite operacii: a) 2a 2 xy ⋅ (−2ax 2 ) : (−axy 2 ) = 1 b) (− x 2 y 3 ) 4 ⋅ ( xy ) 2 = 2 2 v) (−5 x + 6 x − 7 x 3 ) ⋅ 2ax = g) (2 x − 4)(3 − x) =

7. Pretstavi go kako binom vo normalen vid, proizvodot: a) (4a + 3 y ) ⋅ (3 y − 4a) = b) (2 x + 7 y ) 2 = v) 98 2 = 8. Izvr{i gi nazna~enite operacii: a) − (6a 4 b + 4a 2 b) : (−2ab) = b) (a 9 + 1) : (a + 1) =

9. Razlo`i go polinomot na prosti mno`iteli: a) 30a 2 b 2 − 36a 3b 3 − 42a 4 b 2 ; b) 5 x(a − b) − 4 xy(a − b) ;

v) xa + yb + xb + ya .

10. Doka`i deka izrazot 2a 2 + 2b 2 mo`e da se pretstavi kako zbir od dva kvadrata.

74


TEST - 6 1. Odredi go polinomot N , taka {to: N + (a 3 + 2a 2 − 2a − 1) = 2a 3 − 4a 2 + a − 3 .

2. Izvr{i gi nazna~enite operacii: a) − 16 x 8 y 6 : (−4 x 2 y 3 ) = b) (−3xy 3 z ) 4 = v) − 4 x ⋅ (2 x 2 − 3x + 5) = g) (7a + 2) ⋅ (4a 2 − 5a + 3) = 3. Pretstavi go kako polinom vo normalen vid, proizvodot: a) 10,5 ⋅ 9,5 = b) (3x 2 + 5 x) 2 = v) (2a − 3c) 2 =

4. Od monomite: − 2 xy 3 ;−4 xy;− xy 3 i 7 xy a) sostavi polinom b) svedi go vo normalen vid v) odredi go stepenot na polinomot po ( y ) g) odredi go sprotivniot polinom -

5. Presmetaj ja tretata strana na triagolnikot, ako perimetarot e 40a − 3b , a dvete strani se 17a + 3b i 11b − 3a .

75


6. Izvr{i gi nazna~enite operacii: a) − 2a 5 b ⋅ (−3ab 2 ) = 1 b) (−4 x 5 y 3 ) 2 ⋅ (− a 2 x) = 2

v) (2 x − 5) ⋅ (3 + x) = 1 g) − 4 x 2 (− x 3 + 2 x − ) = 2

7. Pretstavi go proizvodot vo normalen vid: a) (5 x − 3 y ) ⋅ (3 y + 5 x) = b) (3a − 5b) 2 = v) 84 2 = 8. Izvr{i gi nazna~enite operacii: 3 a) (9c 4 p 4 − 3c 3 p 5 + 6c 2 p 3 ) : c 2 p 3 = 2

b) ( x 7 + 1) : ( x + 1) =

9. Razlo`i go polinomot na prosti mno`iteli: 5 1 1 b) (a + 2)4 x 2 y − (a + 2)(8 xy 2 ) ; a) x 3 y 4 + 2 x 4 y 2 − 3 x 2 y 2 ; 9 3 3

10. Doka`i go identitetot: (ax + by ) 2 − (ay + bx) 2 = (a 2 − b 2 )( x 2 − y 2 )

76

v) a 2 x − b 2 y − a 2 y − b 2 y .


TEST - 7 1. Izvr{i gi nazna~enite operacii: a) (24 x 5 y 4 − 32 x 2 y 5 + 16 xy ) : 4 xy =

b) ( y 3 − 4 y 2 + 7 y − 6) : ( y 2 − 2 y + 3) =

2. Razlo`i go polinomot na prosti mno`iteli: 1 1 5 a) 2 a 5b + 1 a 4 b 2 − a 3b 3 = 4 10 2

b) 9 x 2 (a − b) + 18 x 3 y (a − b) = v) 2ax + 3ay + 2bx + 3by =

3. Doka`i deka ravenstvoto e to~no: ( x 2 − 2)( x 2 2 x + 1) − x ⋅ ( x 3 − 2 x 2 − 3x + 4) = 2 x 2 − 2 .

4. Od monomite: 2 x 3 y ; − 2 x 2 y 2 ; + xy 3 ; − 4 x 3 y a) sostavi polinom b) svedi go vo normalen vid v) odredi go stepenot na polinomot po (xy ) g) odredi go sprotivniot polinom 5. Zbirot 73x + 20 denari e podelen na tri lica, taka {to liceto A dobilo 31x + 5 denari, liceto B dobilo 20 x − 5 denari, a tretoto lice go dobilo ostatokot. Po kolku denari dobile licata A, B i C.

77


6. Izvr{i gi nazna~enite operacii: a) 24 x 3 y 4 : (3x 2 y ) = b) (−

3 3 3 xy ) = 4

v) − 2ax ⋅ (4 x 2 − 3x + 5) = g) (8 x 3 − 5 x 2 + 4) ⋅ (3x 2 − 2 x) =

7. Pretstavi go kako polinom vo normalen vid, proizvodot: a) 24 ⋅16 = 1 b) ( a + 1) 2 = 2

v) (2 x − 3 y ) 2 = 8. Izvr{i gi nazna~enite operacii: a) (6a 6 − 5,2a 5 + 4a 3 ) : 4a 2 = b) ( x 9 − 1) : ( x − 1) = 9. Razlo`i go polinomot na prosti mno`iteli: a) 1,6a 4 x 3 + 8a 5 y 2 =

b) −12a ( x + y ) + 3ab( x + y ) =

10. Doka`i deka ravenstvoto e to~no: 1 1 3c 2 (2c 3 − 4c 2 + ) − 6(c 5 − 3c 4 + c 2 ) = 6c 4 . 3 6

78

v) 2 p 2 + x − p − 2 px =


TEST - 1 1. Neka ABCDE e pravilen petagolnik. Negovite vnatre{ni agli se __________, nadvore{nite agli se __________, a centralnite agli se __________.

2. Proizvolen pravilen n – agolnik ima _______________ oski na simetrija.

3. Dve figuri koi imaat ednakvi plo{tini se vikaat ___________________________________

4. [ irinata (R - r) na eden kru`en prsten e 1 cm, a negovata plo{tina e 5Ď€ cm 2. Radiusite na dvete koncentri~ni kru`nici se ____________________________________________________

5. Dali postoi pravilen n – agolnik so nadvore{en agol 37O?

Odgovor:

___________________________ , bidej}i ______________________________________

6. Obratnata na Pitagorinata teorema glasi: __________________________________________ ___________________________________________________________________________________

103


7. Konstruiraj otse~ka so dol`ina 13 , ako e dadena edini~na dol`ina 1.

8. Presmetaj ja plo{tinata na eden triagolnik, ako dve negovi strani imaat dol`ini 3 cm i 6 cm, a agolot me|u niv e 60O.

Odgovor: _________________________________ 9. Daden e krug so centar vo O i radius 6 cm. Presmetaj go radiusot na krug so centar vo istata to~ka O, takov {to, plo{tinata na kru`niot prsten e 8 pati pogolema od plo{tinata na toj krug.

Odgovor: _________________________________ 10. Konstruiraj pravoagolen triagolnik ABC so prav agol vo to~kata C, ako se poznati otse~kite c 1 i c 2 na koi visinata spu{tena od praviot agol ja razdeluva hipotenuzata.

104


TEST - 2 1. Neka ABCDEF e pravilen {estagolnik so centar vo to~kata O. Goleminata na agolot BAD e ____________________________________ .

2. Neka aglite na eden tetiven ~etiriagolnik se 60O, 80O, 100O, 120O. Toga{ eden mo`en posledovatelen redosled na goleminite na negovite agli e ____________________________

3. Perimetarot na eden deltoid iznesuva 30 cm, a radiusot na vpi{anata kru`nica e 1,5 cm. Plo{tinata na deltoidot e _________________________________________________________

4. Dol`inata na kru`en lak {to odgovara na centralen agol Îą i radius r e ednakva na _____ ___________________________________________________________________________________

5. Plo{tinata na pravilen {estagolnik so strana 2 cm iznesuva _________________________

6. Da se presmeta plo{tinata na eden triagolnik ako se poznati dol`inite na negovite strani: a=10cm, b=15cm i c=12cm. Re{enie:

105


7. Dve kru`nici so radiusi 6 cm i 10 cm se dopiraat odnadvor. Presmetaj ja dol`inata na nivnata zaedni~ka tangenta, {to e zaklu~ena me|u to~kite na dopir.

Odgovor: _________________________________ 8. Edna kru`na sala so dijametar 15 m pokriena e so parket. Dimenziite na edno par~e parket se 4 cm Ă— 22 cm. Kolku takvi par~iwa pribli`no se potrebni za pokrivawe na salata?

Odgovor: _________________________________ 9. Vo dadenata kru`nica vpi{i deltoid, a potoa vo deltoidot vpi{i kru`nica.

10. Tri strani na eden tangenten ~etiriagolnik imaat dol`ini 12cm, 14cm i 16cm. Odredi gi site mo`ni vrednosti na dol`inite na ~etvrtata strana.

Odgovor: _________________________________

106


TEST - 3 1. Site periferni agli vo edna kru`nica {to se nad ist kru`en lak se ___________________

2. Zbirot na site nadvore{ni agli vo eden konveksen n – agolnik iznesuva ________________

3. Kako }e se promeni plo{tinata na eden triagolnik, ako negovata osnova se namali 6 pati, a visinata se nagolemi 2 pati.

Odgovor : _____________________________________________________________________________

4. Kako }e se promeni plo{tinata na eden krug ako negoviot radius se zgolemi 3 pati?

Odgovor : _____________________________________________________________________________

5. Visinata na eden ramnostran triagolnik so strana 4 cm iznesuva ______________________

6. Plo{tinata na triagolnik so strani a, b i c e dadena so __________________________ , kade ___________________________________________________________________________________

107


7. Dve kru`nici so radiusi 6 cm i 8 cm se se~at, pri {to tangentata na ednata kru`nica vo edna od prese~nite to~ki minuva niz centarot na drugata kru`nica. Presmetaj go rastojanieto me|u centrite na dvete kru`nici.

Odgovor: _________________________________ 8. Presmetaj ja plo{tinata na eden deltoid, ako dvete pomali strani zafa}aat agol od 60O i imaat dol`ina 3 cm, a podolgata dijagonala ima dol`ina 8 cm.

Odgovor: _________________________________ 9. Od kru`en lak od edna kru`nica so radius 20 cm {to odgovara na centralen agol od 72O napravena e kru`nica. Kolkava e plo{tinata na taka dobienata kru`nica?

Odgovor: _________________________________ 10. ^etiri agli na eden petagolnik se ednakvi na 110O, 111O, 112O, 113O. Najdi go pettiot agol na petagolnikot.

Odgovor: _________________________________

108


TEST - 4 1. Neka radiusite na OA i OB na kru`nicata k (O,r) zafa}aat agol Îą. Toga{ agolot pome|u tangentite t a i t b na kru`nicata k vo to~kite A i B e __________________________________

2. Pravilen mnoguagolnik e takov mnoguagolnik na koj __________________________________ ___________________________________________________________________________________

3. Plo{tinata na eden ramnokrak trapez so normalni dijagonali so dol`ini 8 cm iznesuva ___________________________________________________________________________________

4. Dol`inata na kru`en lak iznesuva 4,5 cm, a radiusot na kru`nicata e 6 cm. Plo{tinata na soodvetniot kru`en ise~ok e _____________________________________________________

5. Eden pravilen n – agolnik ima neparen broj oski na simetrija. Dali toj e centralno simetri~en?

Odgovor: _____________________________________________________________________________ 6. Nabroj gi vidovite dijagrami koi gi poznava{: ___________________________________________________________________________________

109


7. Neka triagolnikot ABC e pravoagolen so prav agol vo temeto C. Nad stranite AB, BC i CA od nadvore{nata strana na triagolnikot nacrtani se polukrugovi so dijametri AB, BC i CA. Doka`i deka P AB = P AC + P BC , kade P AB e plo{tina na polukrugot so dijametar AB itn.

8. Najdi ja plo{tinata na eden paralelogram so strani 6 cm i 2 cm, ako agolot me|u osnovite e ednakov na 60O.

Odgovor: _________________________________ 9. Plo{tinata na kru`en ise~ok {to odgovara na centralen agol od 130O e 25 cm2. Presmetaj go radiusot na kru`nicata.

Odgovor: _________________________________ 10. Neka ABCDE e pravilen petagolnik. Presmetaj gi aglite na triagolnikot ACD.

Odgovor: _________________________________

110


TEST - 5 1. Centralen agol {to odgovara na

5 od kru`nicata iznesuva ___________________________ 8

2. Neka dol`inite na stranite na eden ~etiriagolnik se 6 cm, 7 cm, 9 cm i 4 cm. Toga{ eden mo`en posledovatelen redosled na dol`inite na negovite strani e _______________________

3. Radiusot na vpi{anata kru`nica vo pravilen {estagolnik so strana 6 cm iznesuva ___________________________________________________________________________________

4. Kako }e se promeni dol`inata na eden kru`en lak, ako centralniot agol se namali 9 pati, a radiusot se zgolemi 3 pati?

Odgovor: _____________________________________________________________________________ 5. Dijagonalata na eden pravoagolnik iznesuva 10 cm, a ednata strana e dolga 8 cm. Drugata strana ima dol`ina ________________________________________________________________

6. Dva od aglite na eden tetiven ~etiriagolnik se ednakvi na 55 O i 45 O. Najdi gi ostanatite dva agla na ~etiriagolnikot.

Odgovor: ____________________________________________________________________________

111


7. Presmetaj ja plo{tinata na eden pravoagolnik ako edna negova strana ima dol`ina 21 cm, a dijagonalata ima dol`ina 29 cm.

Odgovor: _________________________________ 8. Dol`inata na edna kru`nica e za 5 cm podolga od nejziniot dijametar. Presmetaj ja plo{tinata na krugot.

Odgovor: _________________________________ 9. Najdi go vkupniot broj dijagonali vo konveksen mnoguagolnik so: a) 7 strani, b) 9 strani.

Odgovor: _________________________________

10. Eden zemjodelec posadil 3 dekari so lozje, 2 dekari so praski i 2,5 dekari so kajsii. Pretstavi go toa so sektoren dijagram.

112


TEST - 6 1. Goleminata na eden centralen agol e 146O. Soodvetniot periferen agol e ______________

2. Brojot na site dijagonali vo eden n – agolnik iznesuva ________________________________

3. Plo{tinata na ramnostran triagolnik so strana 4 cm iznesuva ________________________

4. Ako dol`inata na kru`en lak e l, a soodvetniot centralen agol e Îą, toga{ radiusot na kru`nicata iznesuva ______________________________

5. Dijagonalite na eden romb imaat dol`ini 6 cm i 8 cm. Stranata na rombot iznesuva __________________________

6. Plo{tinata na eden mnoguagolnik so perimetar L, vo koj mo`e da se vpi{e kru`nica so radius r, e ednakva na _______________________________________________________________

113


7. Presmetaj ja plo{tinata na ramnokrak trapez so osnovi 2 cm i 10 cm, a krakot ima dol`ina 5 cm.

Odgovor: _________________________________ 8. Eden kai{ povrzuva tri kruga so centri vo to~kite A, B i C so radius 1 cm. Presmetaj ja dol`inata na kai{ot ako se znae deka AB = 3 cm, BC = 4 cm i CA = 5 cm.

Odgovor: _________________________________ 9. Vo deltoid eden od vnatre{nite agli obrazuvan od neednakvi strani e 40O. Odredi gi drugite agli, ako deltoidot e tetiven.

Odgovor: _________________________________ 10. Tri posledovatelni strani na eden tangenten ~etiriagolnik imaat dol`ini 5 cm, 10 cm, 8 cm. Najdi ja dol`inata na ~etvrtata strana.

Odgovor: _________________________________

114


TEST - 1 1. Mno`estvo R od podredenite parovi na relacijata R, go vikame _______________________ 2. Нека е дадено множеството М={1,2,3,4} и во него е зададена релација R на следниот начин: (x,y)∈R ако и само ако |x−y|≤1. Apretstavi ja ovaa relacija so graf. Re{enie:

3. Razmerite 24 : 6 i 16 : x se ednakvi za x = _____________________________________________ 4. Daden e grafikot na funkcijata f = {(−1,−2); (0,0); (1,2); (2,4)(3,6)}. Nejziniot analiti~ki zapis e: f (x) = _______________________________

5. Popolni ja tablicata vo koja x i y se obratnoproporcionalni veli~ini: x

2

y

12

3

6 4

6. Presmetaj ja geometriskata sredina za broevite 8 i 18. Re{enie:

Odgovor: Geometriskata sredina e brojot ____________ . 7. Dadeno e mno`estvoto A = {1,3,5}. Zapi{i go so graf i tabelarno mno`estvoto A2. Odgovor: a)

b)

A2 = {

133


8.

Presekot na dijagonalite na kvadratot le`i vo koordinatniot po~etok. Temeto A ima koordinati (-4,-4). Opredeli gi koordinatite na ostanatite tri temiwa na kvadratot.

9. Vo eden den dvi`eweto na temperaturata na vozduhot bilo zapi{ano so sledniov grafik.

~as

2

4

6

8

10

12

14

16

18

20

22

24

temp. OC Koristej}i go grafikot popolni ja tablicata, a potoa opredeli ja najniskata i najvisokata temperatura na denot: Odgovor:

Тemperaturata bila najvisoka vo _______ ~asot, a najniska vo _______ ~asot.

10. Masata na `elezoto od 120 m3 e 93,6 g. Kolkava e masata na `elezoto od 20 cm3? Re{enie:

Odgovor:

134

Masata na `elezoto od 20 cm3 e ________ g.


TEST - 2 1. Mno`estvoto od podredenite parovi na koi prvata komponenta e od mno`estvoto A, a vtorata od B se vika __________________ na mno`estvoto A i B i se ozna~uva so _________. 2. Vrednostite na apscisata i ordinatata na edna to~ka so edno ime se vikaat _____________ na to~kata. 3. Izrazi ja so formula zavisnosta me|u stranite i perimetarot na kvadrat. Odgovor:

4. Kolku treba da iznesuva h vo ravenstvoto 3 : h = 6 : 8, za ravenstvoto da bide proporcija. Odgovor:

h=_________

5. Popolni ja tablicata vo koja x i y se pravoproporcionalni veli~ini so koeficient na proporcionalnost k = 12 . Odgovor: x

2

3

4

y

6. Opredeli go procentot na sol vo rastvor, ako vo 5 kg rastvor ima 250 g sol. Re{enie:

Odgovor:

Vo rastvorot ima _________% sol.

7. Vo mno`estvoto A = {x x ∈ N i x < 13} zadadena e relacijata R = {( x, y ) x ∈ A i y ∈ A i x ⋅ y = 12} . Пretstavi ja relacijata tabelarno.

Re{enie:

Odgovor:

a) R = ______________________________________________________________ b) Relacijata R e ____________________________________________________

135


8. Vodostojot na rekata od 1 do 6 juni se menuva spored dadenata tablica. Nacrtaj go grafikot na vodostojot na rekata vo ovoj vremenski period.

Datum

1

2

3

4

5

6

Vodostoj

4

0

-2

-3

-5

-6

9. Veli~inite x i y vo dadenata tablica se obratnoproporcionalni. Odredi go koeficientot na proporcionalnosta, a potoa popolni ja tablicata. Odgovor: a) k = ______ ;

b) x

8

y

5

4

1 20

0,5 1 2

10. ^etiri traktori mo`at da izoraat edna niva za 12 dena. Za kolku dena istata niva }e ja izoraat 6 traktori so ista mo}nost. Re{enie:

Re{enie: [ este traktori }e ja izoraat nivata za __________ dena.

136


TEST - 3 1. Mno`estvoto A × A se vika________________________________ i se ozna~uva so __________. 2. Kako od grafot na edna relacija se utvrduva deka taa e simetri~na? Odgovor: _____________________________________________________________________________ 3. Нека grafikot na funkcijata e f = {0,2), (1,3), (2,4), (3,5), (4,6), (5,7)} . Zapi{i ja funkcijata analiti~ki. Odgovor: f (x) = y = 3 iska`ana e _______________________ proporcionalnost me|u veli~ix nata x i y so koeficient na proporcionalnosta k = ______ ____.

4. So formulata

5. Popolni ja tablicata vo koja veli~inite x i y se vo obratna proporcionalnost. x

1

y

18

3

9 6

1

6. Konstruiraj go grafikot na prvata proporcionalnost y = −2 x . Re{enie: x y

137


7. Grafikot na edna funkcija e f = {(−1,1), (−2,4), (−3,0), (0,0), (1,1), (2,4), (3,6)} . Pretstavi ja grafi~ki funkcijata. Re{enie: x y

8. Opredeli ja vrednosta m vo proporcijata (m + 3) : 9 = 14 : 2 . Re{enie: Odgovor:

m = ____________

9. Na slikata grafi~ki e pretstavena zavisnosta na vremeto t i brzinata v na telo koe se dvi`i od mestoto A do mestoto B. So pomo{ na grafikot opredeli so koja brzina }e se dvi`i teloto na patot od A do B, ako mu e potrebno vreme od: a) 1 ~as; b) 2 ~asa; v) 4 ~asa; g) 8 ~asa. Odgovor: a)________km, b)________km, v)________km, g)________km

10. Eden patnik za 11 ~asa }e pomine 55 km. Kolku km pat, patnikot }e pomine za 9 ~asa, ako se dvi`i so ista brzina? Re{enie:

Odgovor:

138

Za 9 ~asa }e pomine ________ km.


TEST - 4 1. Zapi{i gi site podredeni parovi ~ija{to prva komponenta mu pripa|a na mno`estvoto A = {2,5,8}, a vtorata na mno`estvoto B = {a, b} . Odgovor: 2.

Vo grafot docrtaj dve strelki za relacijata da bide relacija za podreduvawe.

3. Dopolni ja re~enicata za tvrdeweto da bide to~no. Za edna relacija R od mno`estvoto A kon mno`estvoto B velime deka e preslikuvawe (funkcija), ako ________________________ ___________________________________________________________________________________

4. Od broevite 3, 5, 9 i 15 sostavi edna proporcija. Odgovor:

6 opredlena e ________________________ proporcionalnost so koefix cient na proporcionalnost k = _________

5. So formulata y =

6. Vo edno oddelenie imalo 30 u~enici, od koi 6 bile odli~ni. Izrazi go brojot na drugite u~enici vo procent. Re{enie:

Odgovor:

Procentot na drugite u~enici bil_________%.

7. Funkcijata f e zadadena so mno`estvoto parovi, {(−4,1), (−3,2), (−2,1), (−1,0), (0,1), (1,2), (2,3)} . Opredeli: a) definiciona oblast (mno`estvo) i b) mno`estvo na vrednosti Odgovor:

a) D = {

b) V = {

139


8. Dadena e funkcijata y = − x + 2, x ∈ R . Popolni ja tablicata i nacrtaj go nejziniot grafik. Re{enie: x

-2

-1

0

1

2

y

9. Opredeli dali se pravoproporcionalni veli~inite dadeni vo tabelata, a potoa opredeli go koeficientot na proporcionalnost ako go ima. x

3

-4

-3

2,5

-6,1

y

9

-12

-9

7,5

-18,3

Odgovor: a) Veli~inite se: _______________ ; b) Koeficient na proporcionalnosta e k = ________ .

10. Edna rabota 72 rabotnika }e ja zavr{at za 45 dena. Za kolku dena istata rabota }e ja zavr{at 60 rabotnici? Re{enie:

Odgovor:

140

60 rabotnici rabotata }e ja zavr{at za ___________ dena.


TEST - 5 1. Dadeno e mno`estvo A × B = {(1,2)(2,3)(3,4)(4,5)(5,6)} . Zapi{i gi na tabelaren na~in mno`estvata: A ={

B={

2. To~kata A (-3,-2) le`i vo _____________________ kvadrat od koordinatnata ramnina.

3. Od mno`estvoto A kon mno`estvoto B so graf zadadena e funkcijata f Od grafot odredi: f (1) = _______ ; f (3) = _____ __; f (4) = _____ ___ Odgovor:

f (1) = _______ ; f (3) = _____ __; f (4) = _____ ___

4. Funkcijata f zadadena e so mno`estvo poredeni parovi {(−5,−2), (−3,0), (−1,2), (1,4), (2,5)} . Pretstavi ja funkcijata so tablica: x y

5. Pravata proporcionalnost na veli~inite x i y dadena e so podredenite parovi (2,8); (3,12); (4,16); (6,24). Odredi go koeficientot na proporcionalnost? Odgovor:

k = _______

141


6. Konstruiraj go grafot na obratna proporcionalnost y =

X

-1

-2

-4

-8

1

2

4

8 . x

8

Y

7.

Vo mno`estvoto M = {A,B,V,G,D,\ ,E} e zadadena relacijata R, “.... e sestra na ....”. a) relacijata pretstavi ja na tabelaren na~in R =________________________________________ b) zapi{i go mno`estvoto na mомчиња A = ________________________________________

8. Vo edno u~ili{te brojot na u~enici ~lenovi na grupite, mladi matemati~ari, mladi fizi~ari i mladi biolozi se odnesuva kako 5 : 4 : 2 soodvetno, a nivniot vkupen broj e 55. Opredeli go brojot na ~lenovite vo sekoja grupa: Re{enie: Odgovor:

matemati~ari: ________;

fizi~ari: ________ ;

biolozi: ________ .

9. Se znae deka veli~inite x i y se obratnoproporcionalni. a) Пopolni ja slednava tabela. x y

1

2

4

8

-1

-2

-4

-2

b) Нapi{i ja formulata {to ja izrazuva proporcionalnosta me|u veli~inite x i y . f ( x) = ___________________________________

10. 32 rabotnika mo`at da asfaltiraat edna ulica za 12 dena. Brojot na rabotnicite se zgolemil za 16. Za kolku dena }e bide zavr{ena istata rabota? Re{enie: Odgovor:

142

Rabotata }e bide zavr{ena za _________ dena.


SODR@INA TEMA I - VEKTORI. TRANSLACIJA 1. 2. 3. 4. 5. 6. 7.

PRAVEC I NASOKA. NASO^ENA OTSE^KA – VEKTOR ....................................................................................... 5 EDNAKVOST NA VEKTORI .................................................................................................................................... 7 SOBIRAWE NA VEKTORI. SVOJSTVA ................................................................................................................. 9 ODZEMAWE NA VEKTORI. MNO@EWE NA VEKTORI SO BROJ ...................................................................... 11 TRANSLACIJA ...................................................................................................................................................... 13 SVOJSTVA NA TRANSLACIJATA ........................................................................................................................ 15 PRIMENA NA TRANSLACIJA .............................................................................................................................. 17

TEST 1 ............................................................................................................................................................................ 19 TEST 2 ............................................................................................................................................................................ 21 TEST 3 ............................................................................................................................................................................ 23 TEST 4 ............................................................................................................................................................................ 25 TEST 5 ............................................................................................................................................................................ 27

TEMA II – STEPENI. KVADRATEN KOREN 1. 2. 3. 4. 5. 6. 7.

POIM ZA STEPEN SO POKAZATEL PRIRODEN BROJ ..................................................................................... 29 MNO@EWE I DELEWE NA STEPENI SO EDNAKVI OSNOVI. STEPENUVAWE NA PROIZVOD. KOLI^NIK I STEPEN .................................................................................. 31 POIM ZA KVADRAT NA RACIONALEN BROJ. POIM ZA KVADRATEN KOREN OD RACIONALEN BROJ ................................................................................... 33 PRESMETUVAWE KVADRATEN KOREN OD RACIONALEN BROJ ..................................................................... 35 POIM ZA IRACIONALEN BROJ ......................................................................................................................... 35 REALNI BROEVI .................................................................................................................................................. 35 PRETSTAVUVAWE NA REALNITE BROEVI NA BROJNA OSKA ....................................................................... 35

TEST 1 ............................................................................................................................................................................ 37 TEST 2 ............................................................................................................................................................................ 38 TEST 3 ............................................................................................................................................................................ 39 TEST 4 ............................................................................................................................................................................ 40 TEST 5 ............................................................................................................................................................................ 41 TEST 6 ............................................................................................................................................................................ 42

TEMA III – POLINOMI 1. 2. 3. 4. 5. 6. 7. 8. 9. 10. 11. 12. 13. 14. 15. 16.

ALGEBARSKI IZRAZ.BROJNA VREDNOST NA IZRAZ (BROJNI IZRAZI. IZRAZI SO PROMENLIVI) ................................................................................................ 43 POIM ZA MONOM. SLI^NI I SPROTIVNI MONOMI .................................................................................... 45 BINOM. TRINOM. POLINOM .............................................................................................................................. 47 STEPEN NA MONOMOT I POLINOMOT ............................................................................................................. 47 SOBIRAWE I ODZEMAWE NA MONOMI ........................................................................................................... 49 SPROTIVNI POLINOMI. OSLOBODUVAWE OD ZAGRADI ............................................................................ 49 SOBIRAWE I ODZEMAWE NA POLINOMI ....................................................................................................... 51 MNO@EWE I DELEWE NA MONOMI .................................................................................................................. 53 STEPENUVAWE NA MONOMI .............................................................................................................................. 53 MNO@EWE NA POLINOM SO MONOM............................................................................................................... 53 MNO@EWE NA POLINOMI ................................................................................................................................ 55 FORMULI ZA SKRATENO MNO@EWE ................................................................................................................ 57 DELEWE NA POLINOM SO MONOM ................................................................................................................... 59 DELEWE NA POLINOM SO POLINOM ............................................................................................................... 59 VIDOVI RACIONALNI IZRAZI ......................................................................................................................... 61 RAZLO@UVAWE NA POLINOMITE NA PROSTI MNO@ITELI ...................................................................... 63

143


TEST 1 TEST 2 TEST 3 TEST 4 TEST 5 TEST 6 TEST 7

............................................................................................................................................................................65 ............................................................................................................................................................................67 ............................................................................................................................................................................69 ............................................................................................................................................................................71 ............................................................................................................................................................................73 ............................................................................................................................................................................75 ............................................................................................................................................................................77

TEMA IV – KRU@NICA I MNOGUAGOLNIK. PLO[ TINA 1. 2. 3. 4. 5. 6. 7. 8. 9. 10. 11. 12.

CENTRALEN AGOL. SVOJSTVA ............................................................................................................................79 PERIFEREN AGOL. TALESOVA TEOREMA .........................................................................................................80 KONSTRUKCIJA NA TANGENTA NA KRU@NICA ...............................................................................................81 TETIVEN ^ETIRIAGOLNIK ................................................................................................................................82 TANGENTEN ^ETIRIAGOLNIK ...........................................................................................................................83 OP[ TO ZA MNOGUAGOLNIKOT ..........................................................................................................................84 PRAVILNI MNOGUAGOLNICI ............................................................................................................................85 OPI[ ANA I VPI[ ANA KRU@NICA ..................................................................................................................86 KONSTRUKCIJA NA NEKOI PRAVILNI MNOGUAGOLNICI ...........................................................................87 PITAGOROVA TEOREMA .......................................................................................................................................88 PRIMENA NA PITAGOROVA TEOREMA ..............................................................................................................89 KONSTRUKCIJA NA TO^KI NA BROJNATA OSKA

13. 14. 15. 16. 17. 18. 19. 20. 21. 22. 23. 24.

KOI ODGOVARAAT NA BROEVITE 2 , 3 , 5 , .. .........................................................................................90 POIM ZA PLO[ TINA............................................................................................................................................91 PLO[ TINA NA PRAVOAGOLNIK .........................................................................................................................92 PLO[ TINA NA PARALELOGRAM .........................................................................................................................93 PLO[ TINA NA TRIAGOLNIK ..............................................................................................................................94 PLO[ TINA NA TRAPEZ ........................................................................................................................................95 PLO[ TINA NA DELTOID .....................................................................................................................................96 PLO[ TINA NA PRAVILEN MNOGUAGOLNIK ....................................................................................................97 DOL@INA NA KRU@NICA....................................................................................................................................98 DOL@INA NA KRU@EN LAK.................................................................................................................................99 PLO[ TINA NA KRUG ........................................................................................................................................... 100 PLO[ TINA NA KRU@EN ISE^OK I KRU@EN PRSTEN ................................................................................. 101 DIJAGRAMI .......................................................................................................................................................... 102

TEST 1 TEST 2 TEST 3 TEST 4 TEST 5 TEST 6

.......................................................................................................................................................................... 103 .......................................................................................................................................................................... 105 .......................................................................................................................................................................... 107 .......................................................................................................................................................................... 109 .......................................................................................................................................................................... 111 .......................................................................................................................................................................... 113

TEMA V – FUNKCIJA. PROPORCIONALNOST 1. 2. 3. 4. 5. 6. 7. 8. 9. 10.

DEKARTOV PROIZVOD NA MNO@ESTVA ........................................................................................................ 115 PRAVOAGOLEN KOORDINATIVEN SISTEM ................................................................................................... 117 RELACIJA ............................................................................................................................................................119 FUNKCIJA ........................................................................................................................................................... 120 ZADAVAWE NA FUNKCII .................................................................................................................................. 121 RAZMER. PROPORCIJA. PRODOL@ENA PROPORCIJA ................................................................................ 123 PRAVA PRAPORCIONALNOST .......................................................................................................................... 125 OBRATNA PROPORCIONALNOST .................................................................................................................... 127 PROSTO TROJNO PRAVILO ..............................................................................................................................129 RABOTA SO PODATOCI ..................................................................................................................................... 131

TEST 1 TEST 2 TEST 3 TEST 4 TEST 5

144

.......................................................................................................................................................................... 133 .......................................................................................................................................................................... 135 .......................................................................................................................................................................... 137 .......................................................................................................................................................................... 139 .......................................................................................................................................................................... 141


Issuu converts static files into: digital portfolios, online yearbooks, online catalogs, digital photo albums and more. Sign up and create your flipbook.